Магнитное поле создается только: 1. Магнитное поле создаётся: а) неподвижными зарядами б)любыми зарядами в)движущимися

Содержание

Контрольная работа 11 кл М п ЭМИ ЭМК ЭМВ

Контрольная работа на тему: «МАГНИТНОЕ ПОЛЕ. ЭЛЕКТРОМАГНИТНАЯ ИНДУКЦИЯ.

ЭЛЕКТРОМАГНИТНЫЕ КОЛЕБАНИЯ И ВОЛНЫ»

ВАРИАНТ 1

ЧАСТЬ А Выберите один верный ответ

1. Магнитное поле создается

1) электрическими зарядами

2) магнитными зарядами

3) движущимися электрическими зарядами

4) любым телом

2. Линии магнитной индукции вокруг проводника с током правильно показаны в случае


1)А 3

2) Б 4) Г

3. Прямолинейный проводник с током находится между полюсами магнита (проводник расположен перпендикулярно плоскости листа, ток течет к читателю). Сила Ампера, действующая на проводник, направлена

1) вправо → 3) вверх ↑

2) влево ← 4) вниз

4. Траектория полета электрона, влетевшего в однородное магнитное поле под углом 60°

1) прямая 3) парабола

2) окружность 4) винтовая линия

5.Какой из ниже перечисленных процессов объясняется явлением электромагнитной индукцией?

1) взаимодействие проводников с током.

  1. отклонение магнитной стрелки при прохождении по про­воду электрического тока.

  2. возникновение электрического тока в замкнутой катуш­ке при увеличении силы тока в катушке, находящейся рядом с ней.

  3. возникновение силы, действующей на прямой проводник с током.

6. Легкое проволочное кольцо подвешено на нити. При вдвигании в кольцо магнита северным полюсом оно будет:

  1. отталкиваться от магнита

  2. притягиваться к магниту

  3. неподвижным

  4. сначала отталкиваться, затем притягиваться

7. На рисунке представлен график зависимости заряда от времени в колебательном контуре. Значения амплитуды заряда и периода его изменения равны

1,5 нКл, 2 мкс

  1. 3 нКл, 4 мкс

  2. 1,5 нКл, 4 мкс

  3. 3 нКл, 2 мкс

ЧАСТЬ В

8. Установите соответствия технических устройств из левого столбца таблицы с физическими явлениями, используемыми в них, в правом столбце.

Устройства

А. электродвигатель

Б. компас

В. гальванометр

Г. МГД – генератор

Явления

  1. действие магнитного поля на постоянный магнит

  2. действие магнитного поля на движущийся электрический заряд

  3. действие магнитного поля на проводник с током

Решите задачи.

9. В однородном магнитном поле движется со скоростью 4 м/с перпендикулярно линиям магнитной индукции провод длиной 1,5м. Модуль вектора индукции магнитного поля равен 50 мТл. Определить ЭДС индукции, которая возникает в проводнике.

10. На какой частоте работает радиопередатчик, излучающий волну длиной 30 м?

ЧАСТЬ С

Решите задачу.

11. По горизонтальным рельсам, расположенным в вертикальном магнитном поле с индукцией 0,01 Тл, скользит проводник длиной 1 м с постоянной скоростью 10 м/с. Концы рельсов замкнуты на резистор сопротивлением 2 Ом. Найдите количество теплоты, которое выделится в резисторе за 4 с. Сопротивлением рельсов и проводника пренебречь.

Контрольная работа на тему: «МАГНИТНОЕ ПОЛЕ. ЭЛЕКТРОМАГНИТНАЯ ИНДУКЦИЯ.

ЭЛЕКТРОМАГНИТНЫЕ КОЛЕБАНИЯ И ВОЛНЫ»

ВАРИАНТ 2

ЧАСТЬ А Выберите один верный ответ

1. Движущийся электрический заряд создает

  1. только электрическое поле

  2. только магнитное поле

  3. как электрическое, так и магнитное поле

  4. только гравитационное поле

2. На рисунке изображен цилиндрический проводник, по которому идет электрический ток. Направление тока указано стрелкой. Как направлен вектор магнитной индукции в точке С?

1)в плоскости чертежа вверх

  1. в плоскости чертежа вниз

  2. от нас перпендикулярно плоскости чертежа

  3. к нам перпендикулярно плоскости чертежа

3. На проводник с током, внесенный в магнитное поле, действует сила, направленная

1)вверх

2)влево

  1. к нам перпендикулярно плоскости чертежа

  2. от нас перпендикулярно плоскости чертежа

4. Скорость электрона направлена перпендикулярно магнитной индукции. Сила Лоренца направлена

1) вправо→

2) влево←

3)вверх ↑

4) вниз

5. Легкое металлическое кольцо подвешено на нити. При вдвигании в кольцо постоянного магнита оно отталкивается от него. Это объясняется

  1. намагничиванием кольца

  2. электризацией кольца

  3. возникновением в кольце индукционного тока

  4. возникновением в магните индукционного тока

6. В проволочное алюминиевое кольцо, висящее на нити, вносят полосовой магнит: сначала южным полюсом, затем северным. Кольцо при этом:

  1. в обоих случаях притянется к магниту

  2. в обоих случаях оттолкнется от магнита

  3. в первом случае притянется, во втором – оттолкнется

  4. в первом случае оттолкнется, во втором – притянется

7.

На рисунке представлен график зависимости силы тока от времени в колебательном контуре. Значения амплитуды силы тока и частоты ее изменения равны

  1. 10мА,8Гц

  2. 10мА,4Гц

  3. 5мА,0,125Гц

4) 5 мА, 0,25 Гц

ЧАСТЬ В

8. Установите соответствия технических устройств из левого столбца таблицы с физическими явлениями, используемыми в них, в правом столбце.

Явления

  1. действие магнитного поля на постоянный магнит

  2. действие магнитного поля на проводник с током

  3. действие магнитного поля на движущийся электрический заряд

Устройства

A. громкоговоритель

Б. электронно-лучевая трубка

B.

амперметр
Г. компас

Решите задачи.

9. В однородном магнитном поле перпендикулярно направлению вектора индукции, модуль которого 0,1 Тл, движется проводник длиной 2 м со скоростью 5 м/с. Определить ЭДС индукции, которая возникает в проводнике.

10. Какова длина волны телевизионного сигнала, если несущая частота равна 50 МГц?

ЧАСТЬ С

11. Решите задачу.

Плоский проволочный виток площадью 1000 см2, имеющий сопротивление 2 Ом, расположен в однородном магнитном поле с индукцией 0,1

Тл таким образом, что его плоскость перпендикулярна линиям магнитной индукции. На какой угол был повернут виток, если при этом по нему прошел заряд 7,5 мКл?

Ответы: 1 2 3 4 5 6 7 8 9 10 11

в-1 3 1 1 4 3 1 3 3132 0,3В 1МГц 20МДж

в-2 3 4 4 3 3 2 3 2321 1В 6м 120°

Магнитное поле | Физика

Магнитное поле — одна из форм материи (отличная от вещества), существующая в пространс­тве, окружающем постоянные магниты, проводники с током и движущиеся заряды. Магнитное поле вместе с электрическим полем образует единое электромагнитное поле.

Магнитное поле не только создается постоянными магнитами, движущимися зарядами и тока­ми в проводниках, но и действует на них же.

Термин «магнитное поле» был введен в 1845 г. М. Фарадеем. К тому времени был уже извес­тен ряд явлений электродинамики, требующих объяснения. К ним относятся, в частности, сле­дующие.

1. Явление взаимодействия постоянных магнитов (установление магнитной стрелки вдоль магнитного меридиана Земли, притяжение разноименных полюсов, отталкивание одноименных), известное с древних времен и систематически исследованное У. Гильбертом (результаты опубли­кованы в 1600 г. в его трактате «О магните, магнитных телах и о большом магните — Земле»).

2. В 1820 г. датский ученый Г. X. Эрстед обнаружил, что магнитная стрелка, помещенная вблизи проводника, по которому течет ток, поворачивается, стремясь расположиться перпендику­лярно проводнику.


3. В том же году французский физик Ампер, которого заинтере­совали опыты Эрстеда, обнаружил взаимодействие двух прямолиней­ных проводников с током. Оказалось, что если токи в проводниках текут в одну сторону, т. е. параллельны, то проводники притягивают­ся (рис. 3.31, а), если в противоположные стороны (т. е. антипарал-лельны), то отталкиваются .

Взаимодействия между проводниками с током, т. е. взаимодейс­твия между движущимися электрическими зарядами, называют маг­нитными, а силы, с которыми проводники с током действуют друг на друга, — магнитными силами.

Согласно теории близкодействия, которой придерживался М. Фа-радей, ток в одном из проводников не может непосредственно влиять на ток в другом проводнике. Аналогично случаю с неподвижными электрическими зарядами, вокруг которых существует электричес­кое поле, был сделан вывод, что в пространстве, окружающем токи, существует магнитное поле, которое действует с некоторой силой на другой проводник с током, помещенный в это поле, либо на посто­янный магнит. В свою очередь, магнитное поле, создаваемое вторым проводником с током, действует на ток в первом проводнике. 
Подобно тому как электрическое поле обнаруживается по его воздействию на пробный заряд, внесенный в это поле, магнитное поле можно обнаружить по ориентирующему действию магнит­ного поля на рамку с током малых (по сравнению с расстояниями, на которых магнитное поле заметно меняется) размеров. Провода, подводящие ток к рамке, следует сплести (или расположить близко друг к другу), тогда результирующая сила, действующая со стороны маг­нитного поля на эти провода, будет равна нулю. Силы же, действующие на такую рамку с током, будут ее поворачивать, так что ее плоскость установится перпендикулярно линиям индукции магнитного поля. В примере, приведенном на рис. 3.32, рамка повернется так, чтобы проводник с током оказался в плоскости рамки. При изменении направления тока в проводнике рамка по­вернется на 180°. В поле между полюсами постоянного магнита рамка повернется плоскостью перпендикулярно магнитным силовым линиям магнита

Магнитное поле — определение, виды

Магнитное поле

Люди только и делают, что говорят про какие-то магнитные бури, привозят магнитики на холодильник, ходят в походы с компасом, который показывает, где север, а где юг. В основе всего этого лежит магнитное поле.

Магнитное поле — это материя, за счет которой осуществляется взаимодействие зарядов.

У нее есть несколько условий для существования:

  • магнитное поле материально, то есть существует независимо от наших знаний о нем;
  • порождается только движущимся электрическим зарядом;
  • обнаружить магнитное поле можно по действию на движущийся электрический заряд (или проводник с током) с некоторой силой;
  • магнитное поле распространяется в пространстве с конечной скоростью, равной скорости света в вакууме.

Магнитное поле создается только движущимся электрическим зарядом? А как же магниты?

Атом состоит из ядра и вращающихся вокруг него электронов. Электроны могут вращаться по разным орбитам. На каждой орбите может находиться по два электрона, которые вращаются в разных направлениях.

Но у некоторых веществ не все электроны парные, и несколько электронов крутятся в одном и том же направлении, такие вещества называются ферромагнетиками. А поскольку электрон — заряженная частица, вращающиеся вокруг атома в одну и ту же сторону электроны создают магнитное поле. Получается миниатюрный электромагнит.

Если атомы вещества расположены в произвольном порядке, поля этих крошечных магнитиков компенсируют друг друга. Но если эти магнитные поля направить в одну и ту же сторону, то они сложатся — и получится магнит.

У любого магнита есть два полюса — северный и южный.

Любое магнитное поле описывается магнитными линиями, которые выходят из северного поля и приходят в южный. Эти линии всегда замкнуты, даже если у них бесконечная длина. Вот так это выглядит:



Как запомнить, что выходят магнитные линии из северного полюса, а приходят в южный?

Все просто — на севере жить никто не хочет. Многие люди переезжают туда, где теплее, зимуют в теплых краях, в общем — стремятся на юг. Магнитные линии тоже.

Северный полюс обозначается латинской буквой N (от английского слова North). А южный — буквой S (от английского слова South).

Наша планета — это один большой магнит. У нее тоже есть северный и южный полюса. Но есть один нюанс — географические полюса отличаются от физических. Да-да, на северном полюсе, который наверху карты, находится южный физический полюс. Ну и наоборот, на южном географическом — северный физический.

Не паникуйте, компас показывает вам географический полюс. Да, компас — это магнитная стрелка, и должен по идее показывать физический полюс, но стрелка окрашена так, чтобы направившись на северный физический полюс, показать южный географический. Чтобы люди не путались.

Опыт Эрстеда

Самое главное экспериментальное доказательство того, что магнитное поле возникает из-за движения зарядов — это опыт Эрстеда. В1820 году Эр­стед опыт­ным пу­тём свя­зал элек­три­че­ст­во и маг­не­тизм с по­мо­щью экс­пе­ри­мен­та с от­кло­не­ни­ем стрел­ки ком­па­са.

Это явление использовали, когда создавали первые ам­пер­мет­ры, так как от­кло­не­ние стрел­ки про­пор­цио­наль­но ве­ли­чи­не то­ка. Оно ле­жит в ос­но­ве лю­бо­го элек­тро­маг­ни­та.

А вот и видео эксперимента:


Источник: YouTube-канал «ШКОЛА ОНЛАЙН»

Магнитное поле. Источники и свойства. Правила и применение

При подключении к двум параллельным проводникам электрического тока, они будут притягиваться или отталкиваться, в зависимости от направления (полярности) подключенного тока. Это объясняется явлением возникновения материи особого рода вокруг этих проводников. Эта материя называется магнитное поле (МП). Магнитной силой называется сила, с которой проводники действуют друг на друга.

Магнитное поле

Теория магнетизма возникла еще в древности, в античной цивилизации Азии. В Магнезии в горах нашли особую породу, куски которой могли притягиваться между собой. По названию места эту породу назвали «магнетиками». Стержневой магнит содержит два полюса. На полюсах особенно сильно обнаруживаются его магнитные свойства.

Магнит, висящий на нитке, своими полюсами будет показывать стороны горизонта. Его полюса будут повернуты на север и юг. На таком принципе действует устройство компаса. Разноименные полюсы двух магнитов притягиваются, а одноименные отталкиваются.

Ученые обнаружили, что намагниченная стрелка, находящаяся возле проводника, отклоняется при прохождении по нему электрического тока. Это говорит о том, что вокруг него образуется МП.

Магнитное поле оказывает влияние на:
  • Перемещающиеся электрические заряды.
  • Вещества, называемые ферромагнетиками: железо, чугун, их сплавы.

Постоянные магниты – тела, имеющие общий магнитный момент заряженных частиц (электронов).

1 — Южный полюс магнита
2 — Северный полюс магнита
3 — МП на примере металлических опилок
4 — Направление магнитного поля

Силовые линии появляются при приближении постоянного магнита к бумажному листу, на который насыпан слой железных опилок. На рисунке четко видны места полюсов с ориентированными силовыми линиями.

Источники магнитного поля
  • Электрическое поле, меняющееся во времени.
  • Подвижные заряды.
  • Постоянные магниты.

С детства нам знакомы постоянные магниты. Они использовались в качестве игрушек, которые притягивали к себе различные металлические детали. Их прикрепляли к холодильнику, они были встроены в различные игрушки.

Электрические заряды, которые находятся в движении, чаще всего имеют больше магнитной энергии, по сравнению с постоянными магнитами.

Свойства
  • Главным отличительным признаком и свойством магнитного поля является относительность. Если неподвижно оставить заряженное тело в некоторой системе отсчета, а рядом расположить магнитную стрелку, то она укажет на север, и при этом не «почувствует» постороннего поля, кроме поля земли. А если заряженное тело начать двигать возле стрелки, то вокруг тела появится МП. В результате становится ясно, что МП формируется только при передвижении некоторого заряда.
  • Магнитное поле способно воздействовать и влиять на электрический ток. Его можно обнаружить, если проконтролировать движение заряженных электронов. В магнитном поле частицы с зарядом отклонятся, проводники с протекающим током будут перемещаться. Рамка с подключенным питанием тока станет поворачиваться, а намагниченные материалы переместятся на некоторое расстояние. Стрелка компаса чаще всего окрашивается в синий цвет. Она является полоской намагниченной стали. Компас ориентируется всегда на север, так как у Земли есть МП. Вся планета – это как большой магнит со своими полюсами.

Магнитное поле не воспринимается человеческими органами, и может фиксироваться только особыми приборами и датчиками. Оно бывает переменного и постоянного вида. Переменное поле обычно создается специальными индукторами, которые функционируют от переменного тока. Постоянное поле формируется неизменным электрическим полем.

Основные правила
Правило буравчика

Силовая линия изображается в плоскости, которая расположена под углом 900 к пути движения тока таким образом, чтобы в каждой точке сила была направлена по касательной к линии.

Чтобы определить направление магнитных сил, нужно вспомнить правило буравчика с правой резьбой.

Буравчик нужно расположить по одной оси с вектором тока, рукоятку вращать таким образом, чтобы буравчик двигался в сторону его направления. В этом случае ориентация линий определится вращением рукоятки буравчика.

Правило буравчика для кольца

Поступательное перемещение буравчика в проводнике, выполненном в виде кольца, показывает, как ориентирована индукция, вращение совпадает с течением тока.

Силовые линии имеют свое продолжение внутри магнита и не могут быть разомкнутыми.

Магнитное поле разных источников суммируются между собой. При этом они создают общее поле.

Магниты с одинаковыми полюсами отталкиваются, а с разными – притягиваются. Значение силы взаимодействия зависит от удаленности между ними. При приближении полюсов сила возрастает.

Параметры магнитного поля
  • Сцепление потоков (Ψ).
  • Вектор магнитной индукции (В).
  • Магнитный поток (Ф).

Интенсивность магнитного поля вычисляется размером вектора магнитной индукции, которая зависит от силы F, и формируется током I по проводнику, имеющему длину l: В = F / (I * l).

Магнитная индукция измеряется в Тесла (Тл), в честь ученого, изучавшего явления магнетизма и занимавшегося их методами расчета. 1 Тл равна индукции магнитного потока силой 1 Н на длине 1 м прямого проводника, находящегося под углом 900 к направлению поля, при протекающем токе в один ампер:

1 Тл = 1 х Н / (А х м).

Правило левой руки

Правило находит направление вектора магнитной индукции.

Если ладонь левой руки разместить в поле, чтобы линии магнитного поля входили в ладонь из северного полюса под 900, а 4 пальца разместить по течению тока, большой палец покажет направление магнитной силы.

Если проводник находится под другим углом, то сила будет прямо зависеть от тока и проекции проводника на плоскость, находящуюся под прямым углом.

Сила не зависит от вида материала проводника и его сечения. Если проводник отсутствует, а заряды движутся в другой среде, то сила не изменится.

При направлении вектора магнитного поля в одну сторону одной величины, поле называется равномерным. Различные среды влияют на размер вектора индукции.

Магнитный поток

Магнитная индукция, проходящая по некоторой площади S и ограниченная этой площадью, является магнитным потоком.

Если площадь имеет наклон на некоторый угол α к линии индукции, магнитный поток снижается на размер косинуса этого угла. Наибольшая его величина образуется при нахождении площади под прямым углом к магнитной индукции:

Ф = В * S.

Магнитный поток измеряется в такой единице, как «вебер», который равен протеканием индукции величиной 1 Тл по площади в 1 м2.

Потокосцепление

Такое понятие применяется для создания общего значения магнитного потока, который создан от некоторого числа проводников, находящихся между магнитными полюсами.

В случае, когда одинаковый ток I протекает по обмотке с количеством витков n, общий магнитный поток, образованный всеми витками, является потокосцеплением.

Потокосцепление Ψ измеряется в веберах, и равно: Ψ = n * Ф.

Магнитные свойства

Магнитная проницаемость определяет, насколько магнитное поле в определенной среде ниже или выше индукции поля в вакууме. Вещество называют намагниченным, если оно образует свое магнитное поле. При помещении вещества в магнитное поле у него появляется намагниченность.

Ученые определили причину, по которой тела получают магнитные свойства. Согласно гипотезе ученых внутри веществ есть электрические токи микроскопической величины. Электрон обладает своим магнитным моментом, который имеет квантовую природу, движется по некоторой орбите в атомах. Именно такими малыми токами определяются магнитные свойства.

Если токи движутся беспорядочно, то магнитные поля, вызываемые ими, самокомпенсируются. Внешнее поле делает токи упорядоченными, поэтому формируется магнитное поле. Это является намагниченностью вещества.

Различные вещества можно разделить по свойствам взаимодействия с магнитными полями. Их разделяют на группы:
  • Парамагнетики – вещества, имеющие свойства намагничивания в направлении внешнего поля, обладающие низкой возможностью магнетизма. Они имеют положительную напряженность поля. К таким веществам относят хлорное железо, марганец, платину и т. д.
  • Ферримагнетики – вещества с неуравновешенными по направлению и значению магнитными моментами. В них характерно наличие некомпенсированного антиферромагнетизма. Напряженность поля и температура влияет на их магнитную восприимчивость (различные оксиды).
  • Ферромагнетики – вещества с повышенной положительной восприимчивостью, зависящей от напряженности и температуры (кристаллы кобальта, никеля и т. д.).
  • Диамагнетики – обладают свойством намагничивания в противоположном направлении внешнего поля, то есть, отрицательное значение магнитной восприимчивости, не зависящая от напряженности. При отсутствии поля у этого вещества не будет магнитных свойств. К таким веществам относятся: серебро, висмут, азот, цинк, водород и другие вещества.
  • Антиферромагнетики – обладают уравновешенным магнитным моментом, вследствие чего образуется низкая степень намагничивания вещества. У них при нагревании осуществляется фазовый переход вещества, при котором возникают парамагнитные свойства. При снижении температуры ниже определенной границы, такие свойства появляться не будут (хром, марганец).
Рассмотренные магнетики также классифицируются еще по двум категориям:
  • Магнитомягкие материалы. Они обладают низкой коэрцитивной силой. При маломощных магнитных полях они могут войти в насыщение. При процессе перемагничивания у них наблюдаются незначительные потери. Вследствие этого такие материалы используются для производства сердечников электрических устройств, функционирующих на переменном напряжении (асинхронный электродвигатель, генератор, трансформатор).
  • Магнитотвердые материалы. Они обладают повышенной величиной коэрцитивной силы. Чтобы их перемагнитить, потребуется сильное магнитное поле. Такие материалы используются в производстве постоянных магнитов.

Магнитные свойства различных веществ находят свое использование в технических проектах и изобретениях.

Магнитные цепи

Объединение нескольких магнитных веществ называется магнитной цепью. Они являются подобием электрических цепей и определяются аналогичными законами математики.

На базе магнитных цепей действуют электрические приборы, индуктивности, трансформаторы. У функционирующего электромагнита поток протекает по магнитопроводу, изготовленному из ферромагнитного материала и воздуху, который не является ферромагнетиком. Объединение этих компонентов является магнитной цепью. Множество электрических устройств в своей конструкции содержат магнитные цепи.

Похожие темы:

Магнитные поля создаются движущимися частицами :: SYL.ru

Магнитное поле создается движущимися электрическими зарядами, поэтому само понятие возникло в электродинамике одновременно с понятием “электрическое поле”. Оно было введено сначала М. Фарадеем, а чуть позже – Дж. Максвеллом, чтобы объяснить, почему электрические заряды имеют такую относительно небольшую дальность взаимодействия.

В эфире

Отцы электродинамики считали, что поле создается путем деформации эфира – невидимой умозрительной среды, заполняющей все сущее (Эйнштейн во время работы над теорией относительности упразднил понятие эфира). Хотя современным людям это и может показаться странным, но до 20 века физики действительно не сомневались в некой субстанции, пронизывающей все сущее. То, как магнитные поля создаются и какова их природа, физики не могли объяснить.

Когда в обиход вошла специальная теория относительности (СТО), а эфир “официально убрали”, пространство стало “пустым”, однако поля даже в вакууме продолжали взаимодействовать, а ведь это невозможно между нематериальными объектами (по крайней мере согласно СТО), поэтому физики сочли нужным присвоить некоторые атрибуты электрическим и магнитным полям. Создаются такие понятия, как масса, импульс и энергия полей.

Свойства магнитного поля

Первое его свойство объясняет природу происхождения: магнитное поле может возникнуть только под воздействием движущихся зарядов (электронов) электрического тока. Силовая характеристика магнитного поля называется магнитной индукцией, она присутствует в любой точке поля.

Воздействие поля распространяется только на движущиеся заряды, магниты и проводники. Оно может быть двух типов: переменного и постоянного. Измерить магнитное поле можно только с помощью специальных приборов, оно не фиксируется человеческими органами чувств (хотя биологи считают, что некоторые животные могут воспринимать изменения в нем). Суть еще одного свойства магнитного поля состоит в том, что оно имеет электродинамическую природу не только потому, что может влиять только на движущиеся заряды, но и потому, что само порождается движением зарядов.

Как увидеть

Хотя органы чувств человека и не могут зафиксировать присутствие магнитного поля, его направление можно определить с помощью намагниченной стрелки. Однако “увидеть” магнитное поле можно с помощью листа бумаги и простых железных опилок. На постоянный магнит необходимо положить лист бумаги, а опилки посыпать сверху, после чего железная стружка выстроится по замкнутым и непрерывным силовым линиям.

Направленность силовых линий определяется с помощью правила правой руки, которое также носит название “правило буравчика”. Если взять проводник в руку таким образом, чтобы большой палец был по направлению тока (ток движется от минуса к плюсу), то остальные пальцы укажут направление силовых линий.

Геомагнетизм

Магнитные поля создаются движущимися зарядами, но тогда какова природа геомагнетизма? Наша планета обладает магнитным полем, которое защищает ее от вредного солнечного излучения, причем диаметр поля в несколько раз превосходит диаметр Земли. По форме оно неоднородно, на “солнечной стороне” сжимается под воздействием солнечного ветра, а с ночной стороны растягивается в виде длинного широкого хвоста.

Считается, что на нашей планете магнитные поля создаются движением токов в ядре, которое состоит из жидкого металла. Это называется “гидромагнитное динамо”. Когда вещество достигает температуры в несколько тысяч градусов по Кельвину, его проводимость становится достаточно высокой, чтобы движения, даже в среде со слабым намагничиванием, начали создавать электрические токи, которые, в свою очередь, и создают магнитные поля.

В локальных областях магнитные поля создаются намагниченными горными породами из верхних слоев планеты, образующих земную кору.

Движение полюсов

С 1885 года началась регистрация движения магнитных полюсов. За последний век южный полюс (полюс в Южном полушарии) переместился на 900 километров, а северный (арктический) магнитный полюс с 1973 года за 11 лет переместился на 120 км, а за следующие десять лет – еще на 150. Согласно последним данным, скорость смещения арктического полюса увеличилась с 10 километров в год до 60.

Хотя ученые знают, как создается магнитное поле Земли, повлиять на движение полюсов не могут и предполагают, что довольно скоро произойдет очередная инверсия. Это естественный процесс, такое на планете не впервые, однако чем подобный процесс обернется для людей – неизвестно.

Из-за чего образуется магнитное поле

Действие магнитного поля распространяется на все виды жизни на Земле и жизни планет. Эта материя, с помощью которой взаимодействуют заряженные частицы. 

Магнит – это предмет, который долгое время находится в одном состоянии, в намагниченном состоянии. С помощью этого свойства такие предметы, как магниты притягивают другие предметы, состоящие из железа и их сплавов. Магниты имеют два полюса – северный и южный, самое сильное магнитное поле располагается около полюсов. 

Магниты бывают натуральными, сделанные из железной руды магнитного железняка. Также магниты бывают искусственными, произведенные человеком. Их делают путем внесения железа в магнитное поле. 

Магнитное поле бывает отрицательным и положительным. Два отрицательных поля и два положительных поля отталкиваются друг от друга, а два поля с разными полюсами будут притягиваться. Это происходит из-за взаимодействия друг с другом магнитных полей. Магнитное поле – вещь не постоянная. Оно может внезапно появиться и внезапно пропасть, все зависит от внешних факторов, влияющих на магнитное поле. 

Элементарные магнитные поля создаются благодаря движению электронов вокруг ядра атома и движению вокруг своей оси. Само магнитное поле образуется благодаря внесению железного предмета во внешнее магнитное поле, тогда элементарные магнитные поля в железном предмете ориентируются во внешнем магнитном поле абсолютно одинаково. После этих небольших преобразований обычный предмет из железа становится магнитом, со своими магнитными полями. 

Действие магнитного поля влияет только на самого себя, а на электрическое поле оно никак не влияет. Есть электрическая заряженная частица, которая непременно движется, вокруг этой частицу и существует магнитное поле. Есть вторая электрическая заряженная частица, вокруг которой также существует магнитное поле. И эти два магнитных поля друг с другом взаимодействуют. 
Действие магнитного поля – это взаимодействия нескольких тел, такие как притягивание и отталкивание. Различаются эти взаимодействия только по интенсивности действия. Например, все электрические двигатели работают по принципу взаимного магнитного отталкивания. 

Наша планета, Земля, как и многие другие планеты, имеет магнитное поле. Магнитное поле Земли возникло из-за того, что наше планета постоянно движется вокруг Солнца и вокруг своей оси. Ядро нашей планеты состоит металла и является проводником электричества. Магнитное поле оказывает благотворное влияние на жизнь целой планеты и взаимодействия около земного пространства. Например, магнитное поле защищает все живое на земле от неблагоприятных воздействий солнца. Также защищает искусственные спутники Земли. Даже красивые полярные сияния вызваны магнитным полем Земли.

1.5Магнитное поле создается не только электрическими токами, но и постоянными

Билет №1

1.1Взаимодействия между проводниками с током, т.е. взаимодействие между направленно

движущимися электрическими зарядами, называются магнитными. Силы, с которыми

проводники с током действуют друг на друга, называются магнитными силами. В

пространстве, окружающем токи, возникает поле, называемое магнитным. Магнитное поле

представляет собой особую форму материи, посредством которой осуществляется

взаимодействие между движущимися электрически заряженными частицами. Свойства:

1) магнитное поле порождается электрическим током.2)магнитное поле обнаруживается

по действию на электрический ток. Подобно электрическому полю, магнитное существует

реально, независимо от нас, от наших знаний о нем. Магнитное поле создается не только

электрическим током, но и постоянными магнитами. 1.2.Векторную характеристику

магнитного поля называют вектором магнитной индукции и обозначают буквой В(вектор).

За направление вектора магнитной индукции принимается направление, которое показывает

северный полюс N магнитной стрелки, свободно устанавливающейся в магнитном поле.

Это направление совпадает с направлением положительной нормали к замкнутому контуру

с током. Правило буравчика: если направление поступательного движения буравчика

совпадает с направлением тока в проводнике, то направление вращения ручки буравчика

указывает направление вектора магнитной индукции. Картину магнитного поля можно

получить если построить так называемые линии магнитной индукции. Линиями магнитной

индукции называют линии, касательные к которым в любой их точке совпадают с вектором

B(вектор) в данной точке поля. Поля с замкнутыми векторными линиями называют вихревым.

Магнитное поле – вихревое поле. Замкнутость линий магнитной индукции представляет

собой фундаментальное свойство магнитного поля. Оно заключается в том что магнитное

поле не имеет источников. Магнитных зарядов, подобных электрическим, в природе не

существует. 1.3Магнитное поле действует на все участки проводника с током. Закон

определяющий силу действующую на отдельный небольшой участок проводника

(элемент тока) был установлен в 1820г Ампером. F=I*|B(вектор)|*дельта l*sina –закон.

Сила Ампера равна произведению модуля силы тока, вектора магнитной индукции,

длины отрезка проводника и синуса угла между направлениями векторов магнитной

индукции и тока. Правило левой руки: если левую руку расположить так, чтобы

перпендикулярная проводнику составляющая вектора магнитной индукции B(вектор)

входила в ладонь, а четыре вытянутых пальца были направлены по направлению тока, то

отогнутый на 90 большой палец укажет направление силы, действующей на отрезок

проводника. За единицу модуля вектора магнитной индукции можно принять магнитную

индукцию однородного поля, в котором на отрезок проводника длиной 1м при силе тока

в нем 1А действует со стороны поля максимальная сила F=1H. Единица магнитной

индукции равна 1H/A*м. 1.4Силу действующую на движущуюся заряженную частицу со

стороны магнитного поля, называют силой Лоренца. Модуль силы Лоренца равен

отношению модуля силы F, действующей на участок проводника дельта l, к числу N

заряженных частиц, упорядоченно движущихся в этом участке проводника.

Fл=F/N=|q|vBsina. Ее направление определяется правилом левой руки: если левую

руку расположить так, чтобы составляющая магнитной индукции B(вектор),

перпендикулярная скорости заряда, входила в ладонь, а четыре вытянух пальца

были направлены по движению положительного заряда, то отогнутый на 90 большой

палец укажет направление действующей на заряд силы Лоренца. Эта сила не совершает

работы(т.к. перпендикулярна скорости частицы), т.е. не меняет кинетическую энергию.

магнитами. Магнитные свойства тела можно объяснить циркулирующими внутри него

токами. Магнитные свойства любого тела определяются замкнутыми электрическими

токами внутри него. Наиболее сильные магнитные поля создают вещества. называемые

ферромагнетиками. При температурах, больших некоторой определенной для данного

ферромагнетика, его ферромагнитные свойства исчезают.(температура Кюри для железа

753). Большое применение получили ферриты- ферромагнитные материалы, не

проводящие электрического тока.

Формула для задачи:

Билет №2

2.1.Явление электромагнитной индукции заключается в возникновении электрического тока в проводящем контуре,

который либо покоится в переменном во времени магнитном поле, либо движется в постоянном магнитном поле,

таким образом, что число линий магнитной индукции, пронизывающих контур, меняется. Это явление было

открыто 29.08.1831. Сначала Фарадеем была открыта электомагнитная индукция в неподвижных относительно

друг друга проводниках при замыкании и размыкании цепи. Затем, поняв, что сближение или удаление

проводников с током должно приводить к такому же результату. В течение одного месяца Фарадей опытным

путем установил все главные особенности явления электромагнитной индукции. В замкнутом проводящем контуре

возникает ток при изменении числа линий магнитной индукции, пронизывающих поверхность, ограниченную этим

контуром. И чем быстрее меняется число линий магнитной индукции, тем больше возникающий индукционный

ток.2.2Для того что бы дать точную количественную формулировку закона электромагнитной индукции Фарадея,

нужно ввести новую величину – поток вектора магнитной индукции. Магнитным потоком Ф (потоком вектора

магнитной индукции) через поверхность площадью S называют величину, равную произведению модуля вектора

магнитной индукции B(вектор) на площадь S и косинус угла a между векторами B и n: Ф=B*S*cosa. Произведение

B*cosa=Bn представляет собой проекцию вектора магнитной индукции на нормаль n(вектор) к плоскости контура,

поэтому Ф=Bn*S. Единицей магнитного потока является вебер. Магнитный поток в 1Bб создается однородным

магнитным полем с индукцией 1Тл через поверхность площдью 1м2, расположенную перпендикулярно вектору

магнитной индукции.2.3Правило Ленца: возникающий в замкнутом контуре индукционный ток свои магнитным

полем противодействует тому изменению магнитного потока, которым он вызван. Применять правило надо так:

Как создаются магнитные поля – Видео и стенограмма урока

Токопроводящие проводники

Эксперимент XIX века показал, что провод, по которому проходит электрический ток, является магнитом. Поскольку все электроны движутся по проводу в одном направлении, вокруг провода создается четко определенное магнитное поле. Сила магнитного поля пропорциональна величине тока, протекающего по проводу. Другими словами, увеличение тока увеличивает силу магнитного поля.Итак, если провода такие магнитные, почему мы не видим, как скрепки и вилки летают по комнате и прилипают к ним? Причина в том, что магнитное поле не очень сильно при нормальном, повседневном уровне электрического тока. Нам нужно будет немного поработать, чтобы превратить проволоку в полезный магнит.

Электромагниты

Поместив ферромагнитные материалы в электрическую катушку, вы можете усилить электромагнит.

Электромагнит – это магнит, который использует электрический ток для создания своего магнитного поля.Это отличается от постоянных магнитов, таких как те, что есть в вашем холодильнике, которые полагаются на магнитные свойства атомов в материале для создания магнитного поля. На данный момент наш электромагнит представляет собой просто провод, но магнитное поле слишком слабое, чтобы сделать что-либо практичное. Однако, если мы согнем провод вокруг и вокруг, чтобы сформировать катушку, магнитные поля петель будут концентрироваться в центре. Чтобы еще больше усилить этот эффект, мы можем намотать несколько слоев проволоки друг на друга. Использование большего количества витков провода увеличивает силу магнитного поля.Это явное улучшение по сравнению с нашим одиночным проводом, которое использовалось ранее, но оно все еще недостаточно прочное, чтобы быть действительно практичным.

Мы можем сделать наш электромагнит в несколько тысяч раз сильнее, поместив сердечник из ферромагнитного материала, такого как железо, в центр катушки. Ферромагнитные материалы содержат так называемые магнитные домены, которые представляют собой области в материале, которые действуют как крошечные магниты. Обычно домены имеют произвольную конфигурацию, и материал не проявляет никакого магнетизма.Однако при воздействии магнитного поля, такого как создаваемое нашей катушкой с проволокой, домены начинают выравниваться, и отдельные магнитные поля объединяются в большее поле.

Степень выравнивания домена зависит от силы магнитного поля, создаваемого катушкой, которое, как мы узнали ранее, может контролироваться величиной тока, протекающего через провод. Не менее важно то, что при отключении тока магнитные домены возвращаются к своей случайной конфигурации, и электромагнит теряет почти весь свой магнетизм.Возможность управлять очень мощным магнитом с помощью переключателя имеет множество практических применений.

Электромагниты в действии

Мы используем электромагниты каждый день, даже не осознавая этого. Их можно найти буквально на тысячах различных устройств, потому что они очень полезны. Например, их можно использовать для подъема стали на свалке металлолома, для звонка в школьный звонок на перемену и даже для левитации высокоскоростных поездов. Знаете ли вы, что в динамиках используются электромагниты? Выступающие пользуются преимуществом того факта, что силой электромагнита можно управлять, регулируя электрический ток.

Источник звука посылает ток на электромагнит в динамике, который управляет звуком, который мы слышим.

Динамик имеет постоянный магнит, установленный на раме, и небольшой электромагнит, прикрепленный к гибкому конусу. Источник звука, например радио, посылает переменный ток на электромагнит, который изменяет то, насколько сильно электромагнит реагирует на магнитное поле постоянного магнита. Это контролирует движение конуса, который производит амплитуду и частоту звука, который мы слышим.Как видите, электромагниты позволяют нам преобразовывать электрический ток в полезную механическую силу, которую можно использовать во всех сферах применения.

Краткое содержание урока

Все магнитные поля создаются движущимися заряженными частицами. Стационарные заряженные частицы не создают магнитных полей. Электромагнит – это магнит, который использует электрический ток для создания магнитного поля. Простейший электромагнит – это просто провод, по которому проходит ток, который создает магнитное поле вокруг провода.Если обернуть провод в катушку, магнитное поле в центре катушки становится сильнее. Добавление ферромагнитного сердечника в центр катушки резко увеличивает напряженность магнитного поля.

Ферромагнитные материалы содержат магнитные домены произвольной формы, которые выравниваются под действием магнитного поля. Выравнивание этих доменов объединяет их индивидуальные магнитные поля в одно сильное поле. Изменение тока в проводе вызывает разную степень выравнивания и, следовательно, общую силу электромагнита.Возможность электрического управления магнетизмом электромагнита привела к множеству практических применений.

Результаты обучения

После просмотра этого урока вы сможете делать следующее:

  • Объяснять, как с помощью электричества создаются магнитные поля.
  • Определить электромагнит
  • Опишите, как добавление ферромагнитного материала к проводу увеличивает напряженность магнитного поля.
  • Приведите примеры повседневного применения электромагнитов

Магнитные поля и линии магнитного поля

Цель обучения

К концу этого раздела вы сможете:

  • Определение магнитного поля и описание силовых линий различных магнитных полей.

Говорят, что в детстве Эйнштейн был очарован компасом, возможно, размышляя о том, как стрелка ощущала силу без прямого физического контакта. Его способность глубоко и ясно мыслить о действиях на расстоянии, особенно о гравитационных, электрических и магнитных силах, позже позволила ему создать свою революционную теорию относительности. Поскольку магнитные силы действуют на расстоянии, мы определяем магнитное поле для представления магнитных сил. Графическое изображение линий магнитного поля очень полезно для визуализации силы и направления магнитного поля.Как показано на Фиг.1, направление линий магнитного поля определяется как направление, в котором указывает северный конец стрелки компаса. Магнитное поле традиционно называют B -field .

Рис. 1. Линии магнитного поля определяются так, чтобы они имели направление, которое указывает маленький компас при размещении в определенном месте. (a) Если для отображения магнитного поля вокруг стержневого магнита используются небольшие компасы, они будут указывать в показанных направлениях: от северного полюса магнита к южному полюсу магнита.(Напомним, что северный магнитный полюс Земли на самом деле является южным полюсом с точки зрения определения полюсов стержневого магнита.) (B) Соединение стрелок дает непрерывные линии магнитного поля. Сила поля пропорциональна близости (или плотности) линий. (c) Если бы можно было исследовать внутреннюю часть магнита, было бы обнаружено, что силовые линии образуют непрерывные замкнутые контуры.

Маленькие компасы, используемые для проверки магнитного поля, его не побеспокоят. (Это аналогично тому, как мы проверяли электрические поля с небольшим пробным зарядом.В обоих случаях поля представляют только объект, создающий их, а не зонд, проверяющий их.) На рисунке 2 показано, как магнитное поле появляется для токовой петли и длинного прямого провода, что можно было бы исследовать с помощью небольших компасов. Небольшой компас, помещенный в эти поля, выровняется параллельно линии поля в своем местоположении, а его северный полюс будет указывать в направлении B . Обратите внимание на символы, используемые для ввода и вывода из бумаги.

Рис. 2. Маленькие компасы можно использовать для картирования полей, показанных здесь.(а) Магнитное поле круговой токовой петли похоже на магнитное поле стержневого магнита. (б) Длинный и прямой провод создает поле с силовыми линиями магнитного поля, образующими кольцевые петли. (c) Когда проволока находится в плоскости бумаги, поле перпендикулярно бумаге. Обратите внимание, что символы, используемые для поля, указывающего внутрь (например, хвоста стрелки), и поля, указывающего наружу (например, наконечника стрелки).

Установление соединений: концепция поля

Поле – это способ отображения сил, окружающих любой объект, которые могут воздействовать на другой объект на расстоянии без видимой физической связи.Поле представляет объект, его генерирующий. Гравитационные поля отображают гравитационные силы, электрические поля отображают электрические силы, а магнитные поля отображают магнитные силы.

Обширные исследования магнитных полей выявили ряд жестких правил. Мы используем линии магнитного поля для представления поля (линии – это графический инструмент, а не физическая сущность сами по себе). Свойства силовых линий магнитного поля можно описать следующими правилами:

  1. Направление магнитного поля касается силовой линии в любой точке пространства.Маленький компас укажет направление линии поля.
  2. Сила поля пропорциональна близости линий. Она точно пропорциональна количеству линий на единицу площади, перпендикулярной линиям (называемой плотностью площади).
  3. Силовые линии магнитного поля никогда не могут пересекаться, а это означает, что поле уникально в любой точке пространства.
  4. Линии магнитного поля непрерывны, образуют замкнутые контуры без начала и конца. Они идут от северного полюса к южному.

Последнее свойство связано с тем, что северный и южный полюса не могут быть разделены. Это явное отличие от силовых линий электрического поля, которые начинаются и заканчиваются на положительных и отрицательных зарядах. Если бы магнитные монополи существовали, то силовые линии магнитного поля начинались бы и заканчивались на них.

Сводка раздела

  • Магнитные поля могут быть наглядно представлены силовыми линиями магнитного поля, свойства которых следующие:
    • Поле касается линии магнитного поля.
    • Напряженность поля пропорциональна линейной плотности.
    • Линии поля не могут пересекаться.
    • Полевые линии представляют собой непрерывные петли.

Концептуальные вопросы

  1. Объясните, почему магнитное поле не может быть уникальным (то есть не иметь единственного значения) в точке пространства, где силовые линии магнитного поля могут пересекаться. (Учитывайте направление поля в такой точке.)
  2. Перечислите сходства силовых линий магнитного и электрического поля.Например, направление поля касается линии в любой точке пространства. Также укажите, чем они отличаются. Например, электрическая сила параллельна силовым линиям электрического поля, тогда как магнитная сила, действующая на движущиеся заряды, перпендикулярна силовым линиям магнитного поля.
  3. Заметив, что силовые линии магнитного поля стержневого магнита напоминают силовые линии пары равных и противоположных зарядов, ожидаете ли вы, что магнитное поле будет быстро уменьшаться в силе по мере удаления от магнита? Это согласуется с вашим опытом работы с магнитами?
  4. Магнитное поле Земли параллельно земле во всех местах? Если нет, то где она параллельна поверхности? Его сила одинакова во всех местах? Если нет, то где оно больше всего?

Глоссарий

магнитное поле:
представление магнитных сил
B -поле:
другой термин для обозначения магнитного поля
силовые линии магнитного поля:
графическое изображение силы и направления магнитного поля
направление силовых линий магнитного поля:
направление, на которое указывает северный конец стрелки компаса

20.1 Магнитные поля, силовые линии и сила – Физика

Задачи обучения раздела

К концу этого раздела вы сможете делать следующее:

  • Обобщите свойства магнитов и опишите, как некоторые немагнитные материалы могут намагничиваться
  • Описывать и интерпретировать рисунки магнитных полей вокруг постоянных магнитов и токоведущих проводов
  • Рассчитайте величину и направление магнитной силы в магнитном поле и силы, действующей на провод с током в магнитном поле.

Поддержка учителей

Поддержка учителей

Цели обучения в этом разделе помогут вашим ученикам овладеть следующими стандартами:

  • (5) Студент знает природу сил в физическом мире.Ожидается, что студент:
    • (G) исследуют и описывают взаимосвязь между электрическими и магнитными полями в таких приложениях, как генераторы, двигатели и трансформаторы.

Кроме того, лабораторное руководство по физике в средней школе рассматривает содержание этого раздела лаборатории под названием «Магнетизм», а также следующие стандарты:

  • (5) Научные концепции. Студент знает природу сил в физическом мире. Ожидается, что студент:
    • (ГРАММ) исследовать и описывать взаимосвязь между электрическими и магнитными полями в таких приложениях, как генераторы, двигатели и трансформаторы.

Раздел Основные термины

Магниты и намагничивание

Люди знали о магнитах и ​​магнетизме тысячи лет.Самые ранние записи относятся к древним временам, особенно в регионе Малой Азии под названием Магнезия – название этого региона является источником таких слов, как магнит . Магнитные породы, найденные в Магнезии, которая сейчас является частью западной Турции, вызвали интерес в древние времена. Когда люди впервые обнаружили магнитные породы, они, вероятно, обнаружили, что некоторые части этих пород притягивают куски железа или других магнитных пород сильнее, чем другие части. Эти области называются полюсами магнита.Магнитный полюс – это часть магнита, которая оказывает наибольшую силу на другие магниты или магнитный материал, например, железо. Например, полюса стержневого магнита, показанного на рисунке 20.2, являются местом сосредоточения скрепок.

Рис. 20.2 Стержневой магнит со скрепками, притянутыми к двум полюсам.

Если стержневой магнит подвешен так, что он свободно вращается, один полюс магнита всегда будет поворачиваться на север, а противоположный полюс – на юг. Это открытие привело к созданию компаса, который представляет собой просто небольшой удлиненный магнит, установленный так, чтобы он мог свободно вращаться.Пример компаса показан на рисунке 20.3. Полюс магнита, направленный на север, называется северным полюсом, а противоположный полюс магнита – южным.

Рис. 20.3 Компас – это удлиненный магнит, установленный в устройстве, которое позволяет магниту свободно вращаться.

Открытие того, что один полюс магнита ориентирован на север, а другой – на юг, позволило людям идентифицировать северный и южный полюса любого магнита. Затем было замечено, что северные полюса двух разных магнитов отталкиваются друг от друга, как и южные полюса.И наоборот, северный полюс одного магнита притягивает южный полюс других магнитов. Эта ситуация аналогична ситуации с электрическим зарядом, когда одинаковые заряды отталкиваются, а разные – притягиваются. В магнитах мы просто заменяем заряд на полюс : полюса отталкиваются, а полюса – притягиваются. Это показано на рисунке 20.4, на котором показано, как сила между магнитами зависит от их взаимной ориентации.

Рис. 20.4 В зависимости от их взаимной ориентации полюса магнита будут притягиваться друг к другу или отталкиваться.

Еще раз рассмотрим тот факт, что полюс магнита, направленный на север, называется северным полюсом магнита. Если противоположные полюса притягиваются, то магнитный полюс Земли, который находится близко к географическому Северному полюсу, должен быть магнитным южным полюсом! Точно так же магнитный полюс Земли, который находится близко к географическому Южному полюсу, должен быть магнитным северным полюсом. Эта ситуация изображена на рис. 20.5, на котором Земля представлена ​​как содержащая гигантский внутренний стержневой магнит с южным магнитным полюсом на географическом Северном полюсе и наоборот.Если бы мы каким-то образом подвесили гигантский стержневой магнит в космосе около Земли, то северный полюс космического магнита был бы притянут к южному полюсу внутреннего магнита Земли. По сути, именно это происходит со стрелкой компаса: ее северный магнитный полюс притягивается к южному полюсу внутреннего магнита Земли.

Рис. 20.5. Землю можно представить как содержащую гигантский магнит, проходящий через ее ядро. Южный магнитный полюс магнита Земли находится на географическом Северном полюсе, поэтому северный полюс магнитов притягивается к Северному полюсу, так северный полюс магнитов получил свое название.Точно так же южный полюс магнитов притягивается к географическому Южному полюсу Земли.

Что произойдет, если разрезать стержневой магнит пополам? Вы получаете один магнит с двумя южными полюсами и один магнит с двумя северными полюсами? Ответ отрицательный: каждая половина стержневого магнита имеет северный и южный полюсы. Вы даже можете продолжить разрезать каждую часть стержневого магнита пополам, и вы всегда получите новый, меньший магнит с двумя противоположными полюсами. Как показано на рисунке 20.6, вы можете продолжить этот процесс вплоть до атомного масштаба, и вы обнаружите, что даже самые маленькие частицы, которые ведут себя как магниты, имеют два противоположных полюса.Фактически, ни в одном эксперименте не было обнаружено никаких объектов с одним магнитным полюсом, от мельчайших субатомных частиц, таких как электроны, до самых больших объектов во Вселенной, таких как звезды. Поскольку магниты всегда имеют два полюса, их называют магнитными диполями – di означает два . Ниже мы увидим, что магнитные диполи обладают свойствами, аналогичными электрическим диполям.

Рис. 20.6. Все магниты имеют два противоположных полюса, от самых маленьких, таких как субатомные частицы, до самых больших, таких как звезды.

Watch Physics

Введение в магнетизм

Это видео представляет интересное введение в магнетизм и обсуждает, в частности, как электроны вокруг своих атомов вносят вклад в наблюдаемые нами магнитные эффекты.

Проверка захвата

К какому магнитному полюсу Земли притягивается северный полюс стрелки компаса?

  1. Северный полюс стрелки компаса притягивается к северному магнитному полюсу Земли, который расположен недалеко от географического Северного полюса Земли.
  2. Северный полюс стрелки компаса притягивается к южному магнитному полюсу Земли, который расположен недалеко от географического Северного полюса Земли.
  3. Северный полюс стрелки компаса притягивается к северному магнитному полюсу Земли, который расположен недалеко от географического Южного полюса Земли.
  4. Северный полюс стрелки компаса притягивается к южному магнитному полюсу Земли, который расположен недалеко от географического Южного полюса Земли.

Только определенные материалы, такие как железо, кобальт, никель и гадолиний, обладают сильными магнитными эффектами.Такие материалы называются ферромагнетиками, от латинского слова ferrum , обозначающего железо. Другие материалы обладают слабыми магнитными эффектами, которые можно обнаружить только с помощью чувствительных инструментов. Ферромагнитные материалы не только сильно реагируют на магниты – так, как железо притягивается к магнитам, – но они также могут намагничиваться сами, то есть их можно заставить быть магнитными или превратить в постоянные магниты (рис. 20.7). Постоянный магнит – это просто материал, который сохраняет свои магнитные свойства в течение длительного времени даже при воздействии размагничивающих воздействий.

Рисунок 20.7 Немагниченный кусок железа помещается между двумя магнитами, нагревается, а затем охлаждается, или просто постукивается в холодном состоянии. Утюг становится постоянным магнитом с выровненными полюсами, как показано: его южный полюс примыкает к северному полюсу исходного магнита, а его северный полюс примыкает к южному полюсу исходного магнита. Обратите внимание, что силы притяжения создаются между центральным магнитом и внешними магнитами.

Когда магнит приближается к ранее немагниченному ферромагнитному материалу, он вызывает локальное намагничивание материала с противоположными полюсами, расположенными ближе всего, как показано на правой стороне рисунка 20.7. Это вызывает силу притяжения, поэтому немагнитное железо притягивается к магниту.

То, что происходит в микроскопическом масштабе, показано на Рисунке 7 (а). Области внутри материала, называемые доменами, действуют как маленькие стержневые магниты. Внутри доменов выровнены магнитные полюса отдельных атомов. Каждый атом действует как крошечный стержневой магнит. В немагнитном ферромагнитном объекте домены имеют небольшие размеры и ориентированы случайным образом. В ответ на внешнее магнитное поле домены могут вырасти до миллиметра, выравниваясь, как показано на рисунке 7 (b).Это индуцированное намагничивание можно сделать постоянным, если материал нагреть, а затем охладить, или просто постучать в присутствии других магнитов.

Рис. 20.8 (a) Немагнитный кусок железа или другого ферромагнитного материала имеет произвольно ориентированные домены. (б) При намагничивании внешним магнитом домены демонстрируют большее выравнивание, и некоторые из них растут за счет других. Отдельные атомы выровнены внутри доменов; каждый атом действует как крошечный стержневой магнит.

И наоборот, постоянный магнит можно размагнитить сильными ударами или нагреванием в отсутствие другого магнита.Повышенное тепловое движение при более высокой температуре может нарушить и изменить ориентацию и размер доменов. Для ферромагнитных материалов существует четко определенная температура, называемая температурой Кюри, выше которой они не могут намагничиваться. Температура Кюри для железа составляет 1043 К (770 ° C ° C), что намного выше комнатной температуры. Есть несколько элементов и сплавов, которые имеют температуру Кюри намного ниже комнатной температуры и являются ферромагнитными только ниже этих температур.

Snap Lab

Магниты на холодильник

Мы знаем, что подобные магнитные полюса отталкиваются, а разные полюса притягиваются. Посмотрим, сможете ли вы показать это на примере двух магнитов на холодильник. Прилипнут ли магниты, если их перевернуть? Почему они вообще прилепляются к дверце холодильника? Что вы можете сказать о магнитных свойствах дверцы холодильника возле магнита? Магниты на холодильник прилипают к металлическим или пластиковым ложкам? Прилипают ли они ко всем типам металла?

Поддержка учителя
Поддержка учителя

Если держать магнит рядом с немагнитным ферромагнитным материалом, ферромагнитный материал магнитно поляризуется, что приводит к ориентации атомных магнитных диполей по направлению к внешнему магниту.Это похоже на электрическую поляризацию. Таким образом, ферромагнитный материал намагничивается в присутствии внешнего магнита, и два магнита притягиваются друг к другу. Чтобы магнит прилипал к дверце холодильника, дверца должна содержать какой-то ферромагнитный материал. Магниты будут прилипать к ложкам из железа, например к ложкам с железом, но не к ложкам из цветных металлов, таким как ложки из алюминия или серебра, и не будут прилипать к магниту. Магниты также не будут прилипать к пластиковым ложкам.

Проверка захвата

У вас есть один магнит с обозначенными северным и южным полюсами.Как вы можете использовать этот магнит для определения северного и южного полюсов других магнитов?

  1. Если северный полюс известного магнита отталкивается полюсом неизвестного магнита при приближении их, этот полюс неизвестного магнита является его северным полюсом; в противном случае это его южный полюс.
  2. Если северный полюс известного магнита притягивается к полюсу неизвестного магнита при приближении их, этот полюс неизвестного магнита является его северным полюсом; в противном случае это его южный полюс.

Магнитные поля

Таким образом, мы увидели, что силы могут применяться между магнитами, а также между магнитами и ферромагнитными материалами без какого-либо контакта между объектами.Это напоминает электрические силы, которые действуют на расстоянии. Электрические силы описываются с использованием концепции электрического поля, которое представляет собой силовое поле вокруг электрических зарядов, которое описывает силу, действующую на любой другой заряд, помещенный в это поле. Точно так же магнит создает вокруг себя магнитное поле, которое описывает силу, действующую на другие магниты, помещенные в это поле. Как и в случае с электрическими полями, графическое представление силовых линий магнитного поля очень полезно для визуализации силы и направления магнитного поля.

Как показано на рисунке 20.9, направление силовых линий магнитного поля определяется как направление, в котором указывает северный полюс стрелки компаса. Если вы поместите компас рядом с северным полюсом магнита, северный полюс стрелки компаса будет отталкиваться и указывать в сторону от магнита. Таким образом, силовые линии магнитного поля направлены от северного полюса магнита к его южному полюсу.

Рисунок 20.9 Черные линии представляют силовые линии магнитного поля стержневого магнита.Линии поля указывают в направлении, в котором будет указывать северный полюс небольшого компаса, как показано слева. Силовые линии магнитного поля никогда не прекращаются, поэтому силовые линии фактически проникают в магнит, образуя полные петли, как показано справа.

Силовые линии магнитного поля можно нанести на карту с помощью небольшого компаса. Компас перемещается от точки к точке вокруг магнита, и в каждой точке проводится короткая линия в направлении стрелки, как показано на рисунке 20.10. Соединение линий вместе показывает путь линии магнитного поля.Еще один способ визуализировать силовые линии магнитного поля – это рассыпать железные опилки вокруг магнита. Опилки будут ориентироваться вдоль силовых линий магнитного поля, образуя узор, подобный изображенному справа на рис. 20.10.

Виртуальная физика

Использование компаса для построения карты магнитного поля

Эта симуляция представляет вам стержневой магнит и небольшой компас. Начните с перетаскивания компаса вокруг стержневого магнита, чтобы увидеть, в каком направлении направлено магнитное поле.Обратите внимание, что сила магнитного поля представлена ​​яркостью значков магнитного поля в сетке вокруг магнита. Используйте измеритель магнитного поля, чтобы проверить напряженность поля в нескольких точках вокруг стержневого магнита. Вы также можете изменить полярность магнита или поместить Землю на изображение, чтобы увидеть, как компас ориентируется.

Проверка захвата

С помощью ползунка в правом верхнем углу окна моделирования установите напряженность магнитного поля на 100 процентов.Теперь используйте измеритель магнитного поля, чтобы ответить на следующий вопрос: где магнитное поле самое сильное, а где самое слабое возле магнита? Не забудьте проверить стержневой магнит изнутри.

  1. Магнитное поле самое сильное в центре и самое слабое между двумя полюсами сразу за стержневым магнитом. Силовые линии магнитного поля наиболее плотные в центре и наименее плотные между двумя полюсами сразу за стержневым магнитом.
  2. Магнитное поле самое сильное в центре и самое слабое между двумя полюсами сразу за стержневым магнитом.Линии магнитного поля наименее плотны в центре и наиболее плотны между двумя полюсами сразу за стержневым магнитом.
  3. Магнитное поле самое слабое в центре и самое сильное между двумя полюсами сразу за стержневым магнитом. Силовые линии магнитного поля наиболее плотные в центре и наименее плотные между двумя полюсами сразу за стержневым магнитом.
  4. Магнитное поле самое слабое в центре и самое сильное между двумя полюсами сразу за стержневым магнитом, а силовые линии магнитного поля наименее плотные в центре и самые плотные между двумя полюсами сразу за стержневым магнитом.

Рисунок 20.10 Силовые линии магнитного поля можно нарисовать, перемещая небольшой компас от точки к точке вокруг магнита. В каждой точке проведите короткую линию в направлении стрелки компаса. Соединение точек вместе показывает путь линий магнитного поля. Другой способ визуализировать силовые линии магнитного поля – это рассыпать железные опилки вокруг магнита, как показано справа.

Когда два магнита сближаются, силовые линии магнитного поля возмущаются, как это происходит с силовыми линиями электрического поля, когда два электрических заряда сближаются.Соединение двух северных полюсов или двух южных полюсов вызовет отталкивание, и силовые линии магнитного поля будут отклоняться друг от друга. Это показано на рисунке 20.11, где показаны силовые линии магнитного поля, созданные двумя близко расположенными северными полюсами стержневого магнита. Когда противоположные полюса двух магнитов сводятся вместе, силовые линии магнитного поля соединяются и становятся более плотными между полюсами. Эта ситуация показана на рисунке 20.11.

Рис. 20.11 (a) Когда два северных полюса сближаются, силовые линии магнитного поля отталкиваются друг от друга, и два магнита испытывают силу отталкивания.То же самое происходит, если два южных полюса сближаются. (b) Если противоположные полюса сближаются, силовые линии магнитного поля между полюсами становятся более плотными, и магниты испытывают силу притяжения.

Подобно электрическому полю, магнитное поле сильнее там, где линии более плотные. Таким образом, между двумя северными полюсами на рисунке 20.11 магнитное поле очень слабое, потому что плотность магнитного поля почти равна нулю. Компас, помещенный в эту точку, по сути, будет свободно вращаться, если мы не будем учитывать магнитное поле Земли.Напротив, силовые линии магнитного поля между северным и южным полюсами на рисунке 20.11 очень плотные, что указывает на то, что магнитное поле в этой области очень сильное. Компас, помещенный здесь, быстро выровнялся бы с магнитным полем и указывал бы на южный полюс справа.

Поддержка учителей

Поддержка учителей
Предупреждение о неправильном представлении

Плотность силовых линий магнитного поля на рисунке 20.11 указывает величину силы, которая будет приложена к небольшому испытательному магниту, помещенному в это поле.Плотность не указывает силу между двумя магнитами, создающими поле. Величина силы между двумя магнитами одинакова в обоих случаях на рисунке 20.11. Это можно понять, представив, что вы помещаете один из магнитов в поле другого магнита. Эта ситуация симметрична: магнитные поля выглядят одинаково – за исключением направления – для обеих ситуаций, показанных на рисунке 20.11. Поскольку магниты имеют одинаковую силу, они возмущают магнитное поле противоположного магнита, поэтому магнитное поле необходимо исследовать с помощью небольшого магнитного поля, такого как компас.

Обратите внимание, что магниты – не единственное, что создает магнитные поля. В начале девятнадцатого века люди обнаружили, что электрические токи вызывают магнитные эффекты. Первое важное наблюдение было сделано датским ученым Гансом Кристианом Эрстедом (1777–1851), который обнаружил, что стрелка компаса отклоняется проводом с током. Это было первое существенное свидетельство того, что движение электрических зарядов имеет какую-либо связь с магнитами. Электромагнит – это устройство, которое использует электрический ток для создания магнитного поля.Эти временно индуцированные магниты называются электромагнитами. Электромагниты используются во всем: от крана для разборки, который поднимает сломанные автомобили, до управления пучком ускорителя частиц с окружностью 90 км и магнитов в машинах для медицинской визуализации (см. Рис. 20.12).

Рисунок 20.12 Прибор для магнитно-резонансной томографии (МРТ). В устройстве используется электромагнит с цилиндрической катушкой для создания основного магнитного поля. Пациент проходит в туннель на каталке.(предоставлено Биллом МакЧесни, Flickr)

Магнитное поле, создаваемое электрическим током в длинном прямом проводе, показано на рисунке 20.13. Силовые линии магнитного поля образуют концентрические круги вокруг провода. Направление магнитного поля можно определить с помощью правила правой руки . Это правило проявляется в нескольких местах при изучении электричества и магнетизма. Применительно к прямому токонесущему проводу правило правой руки гласит, что когда большой палец правой руки направлен в направлении тока, магнитное поле будет в том направлении, в котором изгибаются ваши пальцы правой руки, как показано на рисунке 20.13. Если провод очень длинный по сравнению с расстоянием х от провода, сила магнитного поля B будет равна

. B прямой = μ0I2πrB прямой = μ0I2πr

20,1

, где I – ток в проводе в амперах. Единицей измерения магнитного поля в системе СИ является тесла (Тл). Символ μ0μ0 – читается как «мю-ноль» – это константа, называемая «проницаемостью свободного пространства», и задается как

. μ0 = 4π × 10−7T⋅m / A. μ0 = 4π × 10−7T⋅m / A.

20,2

Рисунок 20.13 На этом изображении показано, как использовать правило правой руки для определения направления магнитного поля, создаваемого током, протекающим по прямому проводу. Направьте большой палец правой руки в направлении тока, и магнитное поле будет в том направлении, в котором изгибаются ваши пальцы.

Watch Physics

Магнитное поле, создаваемое электрическим током

В этом видео описывается магнитное поле, создаваемое прямым проводом с током. Он переходит к правилу правой руки для определения направления магнитного поля, а также представляет и обсуждает формулу для силы магнитного поля, создаваемого прямым проводом с током.

Проверка захвата

Длинный прямой провод кладут на столешницу, и электрический ток течет по нему справа налево. Если вы посмотрите на конец провода с левого конца, магнитное поле движется по часовой стрелке или против часовой стрелки?

  1. Если направить большой палец правой руки в направлении, противоположном току, пальцы правой руки будут изгибаться против часовой стрелки, поэтому магнитное поле будет направлено против часовой стрелки.
  2. Если направить большой палец правой руки в направлении, противоположном току, пальцы правой руки будут изгибаться по часовой стрелке, поэтому магнитное поле будет направлено по часовой стрелке.
  3. Если направить большой палец правой руки в направлении тока, пальцы правой руки будут сгибаться против часовой стрелки, поэтому магнитное поле будет направлено против часовой стрелки.
  4. Если направить большой палец правой руки в направлении тока, пальцы правой руки будут изгибаться по часовой стрелке, поэтому магнитное поле будет направлено по часовой стрелке.

Теперь представьте, что наматывается проволока на цилиндр, после чего цилиндр снят. В результате получается катушка с проволокой, как показано на рисунке 20.14. Это называется соленоидом. Чтобы найти направление магнитного поля, создаваемого соленоидом, примените правило правой руки к нескольким точкам катушки. Вы должны убедиться, что внутри катушки магнитное поле направлено слева направо. Фактически, еще одно применение правила правой руки – сгибать пальцы правой руки вокруг катушки в направлении, в котором течет ток. Затем ваш большой палец правой руки указывает в направлении магнитного поля внутри катушки: в данном случае слева направо.

Рисунок 20.14 Катушка с проводом, через которую проходит ток, как показано, создает магнитное поле в направлении красной стрелки.

Каждая петля из проволоки создает магнитное поле внутри соленоида. Поскольку силовые линии магнитного поля должны образовывать замкнутые петли, силовые линии замыкают петлю за пределами соленоида. Силовые линии магнитного поля внутри соленоида намного плотнее, чем вне соленоида. Результирующее магнитное поле очень похоже на магнитное поле стержневого магнита, как показано на рисунке 20.15. Напряженность магнитного поля внутри соленоида

. Bsolenoid = μ0NIℓ, Bsolenoid = μ0NIℓ,

20,3

, где N – количество витков в соленоиде, а ℓℓ – длина соленоида.

Рисунок 20.15 Железные опилки показывают картину магнитного поля вокруг (а) соленоида и (б) стержневого магнита. Картины полей очень похожи, особенно возле концов соленоида и стержневого магнита.

Виртуальная физика

Электромагниты

Используйте это моделирование для визуализации магнитного поля, созданного соленоидом.Обязательно щелкните вкладку с надписью «Электромагнит». Вы можете пропустить через соленоид переменный или постоянный ток, выбрав соответствующий источник тока. Используйте измеритель поля для измерения силы магнитного поля, а затем измените количество витков в соленоиде, чтобы увидеть, как это влияет на напряженность магнитного поля.

Проверка захвата

Выберите аккумулятор в качестве источника тока и установите количество витков на четыре. С ненулевым током, протекающим через соленоид, измерьте напряженность магнитного поля в точке.Теперь уменьшите количество проволочных петель до двух. Как изменится напряженность магнитного поля в выбранной вами точке?

  1. При уменьшении количества витков с четырех до двух напряженность магнитного поля не изменится.
  2. Напряженность магнитного поля уменьшается до половины своего исходного значения, когда количество витков уменьшается с четырех до двух.
  3. Напряженность магнитного поля увеличивается вдвое от исходного значения, когда количество витков уменьшается с четырех до двух.
  4. Напряженность магнитного поля увеличивается в четыре раза от исходного значения, когда количество витков уменьшается с четырех до двух.

Магнитная сила

Если движущийся электрический заряд, то есть электрический ток, создает магнитное поле, которое может воздействовать на другой магнит, то по третьему закону Ньютона должно быть верно обратное. Другими словами, заряд, движущийся через магнитное поле, создаваемое другим объектом, должен испытывать силу – и это именно то, что мы находим.В качестве конкретного примера рассмотрим рисунок 20.16, на котором показан заряд q , движущийся со скоростью v → v → через магнитное поле B → B → между полюсами постоянного магнита. Величина F силы, испытываемой этим зарядом, равна

. F = qvBsinθ, F = qvBsinθ,

20,4

где θθ – угол между скоростью заряда и магнитным полем.

Направление силы может быть найдено с помощью другой версии правила правой руки: сначала мы соединяем хвосты вектора скорости и вектора магнитного поля, как показано на шаге 1 рисунка 20.16. Затем мы сгибаем пальцы правой руки от v → v → к B → B →, как показано в шаге (2) рисунка 20.16. Направление, в котором указывает большой палец правой руки, – это направление силы. Для заряда на рис. 20.16 мы обнаруживаем, что сила направлена ​​внутрь страницы.

Обратите внимание, что множитель sinθsinθ в уравнении F = qvBsinθF = qvBsinθ означает, что к заряду, движущемуся параллельно магнитному полю, приложена нулевая сила, поскольку θ = 0θ = 0 и sin0 = 0sin0 = 0. Максимальная сила, которую может испытывать заряд, – это когда он движется перпендикулярно магнитному полю, потому что θ = 90 ° θ = 90 °. и sin90 ° = 1.sin90 ° = 1.

Рис. 20.16 (а) Протон движется в однородном магнитном поле. (б) Используя правило правой руки, обнаруживается, что сила, действующая на протон, направлена ​​внутрь страницы.

Ссылки на физику

Магнитогидродинамический привод

В романе Тома Клэнси о холодной войне «Охота за Красный Октябрь» Советский Союз построил подводную лодку (см. Рис. 20.17) с магнитогидродинамическим приводом, который был настолько бесшумным, что его невозможно было обнаружить. надводные корабли. Единственная возможная цель создания такой подводной лодки заключалась в том, чтобы дать Советскому Союзу возможность первого удара, потому что эта подводная лодка могла подкрасться к побережью Соединенных Штатов и запустить баллистические ракеты, уничтожая ключевые военные и правительственные объекты, чтобы предотвратить американскую контратаку. .

Рисунок 20.17 Российская подводная лодка с баллистическими ракетами класса «Тайфун», на которой базировалась вымышленная подводная лодка «Красный Октябрь».

Магнитогидродинамический привод должен быть бесшумным, поскольку в нем нет движущихся частей. Вместо этого он использует силу, испытываемую заряженными частицами, движущимися в магнитном поле. Основная идея такого привода изображена на рис. 20.18. Соленая вода течет по каналу, идущему от носа к корме подводной лодки. Магнитное поле прикладывается горизонтально к каналу, а напряжение прикладывается к электродам в верхней и нижней части канала, чтобы направить нисходящий электрический ток через воду.Носителями заряда являются положительные ионы натрия и отрицательные ионы хлора соли. Используя правило правой руки, обнаруживается, что сила, действующая на носители заряда, направлена ​​к задней части судна. Ускоренные заряды сталкиваются с молекулами воды и передают свой импульс, создавая струю воды, которая вылетает из задней части канала. По третьему закону Ньютона на сосуд действует сила равной величины, но в противоположном направлении.

Рис. 20.18 Схематическое изображение магнитогидродинамического привода, показывающее водный канал, направление тока, направление магнитного поля и результирующую силу.

К счастью для всех, оказалось, что такая силовая установка не очень практична. Некоторые предварительные расчеты показывают, что для питания подводной лодки потребуются либо чрезвычайно высокие магнитные поля, либо чрезвычайно высокие электрические токи для получения разумной тяги. Кроме того, прототипы магнитогидродинамических приводов показывают, что они совсем не бесшумны. Электролиз, вызванный пропусканием тока через соленую воду, создает пузырьки водорода и кислорода, что делает эту двигательную установку довольно шумной.Система также оставляет след из хлорид-ионов и хлоридов металлов, который можно легко обнаружить, чтобы определить местонахождение подводной лодки. Наконец, ионы хлора чрезвычайно реактивны и очень быстро разъедают металлические детали, такие как электрод или сам водяной канал. Таким образом, Красный Октябрь остается в сфере фантастики, но его физика вполне реальна.

Проверка захвата

Представьте себе лодку, приводимую в движение силой заряженных частиц, движущихся в магнитном поле. Если магнитное поле направлено вниз, в каком направлении должен течь ток заряженных частиц, чтобы получить силу, направленную назад?

  1. Течение должно течь вертикально сверху вниз, если смотреть сзади лодки.
  2. Течение должно течь вертикально снизу вверх, если смотреть сзади лодки.
  3. Течение должно течь горизонтально слева направо, если смотреть сзади лодки.
  4. Течение должно течь горизонтально справа налево, если смотреть сзади лодки.

Вместо одиночного заряда, движущегося в магнитном поле, рассмотрим теперь постоянный ток I , движущийся по прямому проводу.Если мы поместим этот провод в однородное магнитное поле, как показано на рисунке 20.19, какова сила, действующая на провод или, точнее, на электроны в проводе? Электрический ток включает в себя движущиеся заряды. Если заряды q перемещаются на расстояние ℓℓ за время t , то их скорость будет v = ℓ / t.v = / t. Подставляя это в уравнение F = qvBsinθF = qvBsinθ, получаем

F = q (ℓt) Bsinθ = (qt) ℓBsinθ.F = q (ℓt) Bsinθ = (qt) ℓBsinθ.

20,5

Коэффициент q / t в этом уравнении – не что иное, как ток в проводе.Таким образом, используя I = q / tI = q / t, получаем

F = IℓBsinθ (1.4). F = IℓBsinθ (1.4).

20,6

Это уравнение дает силу, действующую на прямой провод с током длиной в магнитном поле с напряженностью B . Угол θθ – это угол между вектором тока и вектором магнитного поля. Обратите внимание, что ℓℓ – это длина провода, находящегося в магнитном поле, для которого θ ≠ 0, θ ≠ 0, как показано на рисунке 20.19.

Направление силы определяется так же, как и для одиночного заряда.Согните пальцы правой руки от вектора для I к вектору для B , а большой палец правой руки будет указывать в направлении силы, действующей на провод. Для провода, показанного на рис. 20.19, сила направлена ​​внутрь страницы.

Рисунок 20.19 Прямой провод, по которому течет ток I в магнитном поле B . Сила, приложенная к проволоке, направлена ​​внутрь страницы. Длина ℓℓ – это длина провода, равная в магнитном поле.

В этом разделе вы могли заметить симметрию между магнитными и электрическими эффектами.Все эти эффекты подпадают под понятие электромагнетизма, которое является исследованием электрических и магнитных явлений. Мы видели, что электрические заряды создают электрические поля, а движущиеся электрические заряды создают магнитные поля. Магнитный диполь создает магнитное поле, и, как мы увидим в следующем разделе, движущиеся магнитные диполи создают электрическое поле. Таким образом, электричество и магнетизм – два тесно связанных и симметричных явления.

Рабочий пример

Траектория электрона в магнитном поле

Протон входит в область постоянного магнитного поля, как показано на рисунке 20.20. Магнитное поле выходит из страницы. Если электрон движется со скоростью 3,0 × 106 м / с3,0 × 106 м / с, а напряженность магнитного поля составляет 2,0 Тл, каковы величина и направление силы, действующей на протон?

Рис. 20.20. Протон попадает в область однородного магнитного поля. Магнитное поле исходит из страницы – кружки с точками представляют наконечники векторных стрелок, выходящих из страницы.

Стратегия

Используйте уравнение F = qvBsinθF = qvBsinθ, чтобы найти величину силы, действующей на протон.Угол между векторами магнитного поля и вектором скорости протона составляет 90 ° .90 °. Направление силы можно найти с помощью правила правой руки.

Решение

Заряд протона q = 1,60 · 10−19Cq = 1,60 · 10−19C. Ввод этого значения, заданной скорости и напряженности магнитного поля в уравнение F = qvBsinθF = qvBsinθ дает

F = qvBsinθ = (1,60 × 10−19C) (3,0 × 106 м / с) (2,0T) sin (90 °) = 9,6 × 10−13N. F = qvBsinθ = (1,60 × 10−19C) (3,0 × 106 м / с) (2..

Обсуждение

Это похоже на очень маленькую силу. Однако масса протона составляет 1,67 × 10–27 кг, 1,67 × 10–27 кг, поэтому его ускорение равно a = Fm = 9,6 × 10–13N1,67 × 10–27 кг = 5,7 × 1014 м / с2a = Fm = 9,6. × 10–13N1,67 × 10–27 кг = 5,7 × 1014 м / с2, или примерно в десять тысяч миллиардов раз больше ускорения свободного падения!

Мы обнаружили, что начальное ускорение протона, когда он входит в магнитное поле, направлено вниз в плоскости страницы. Обратите внимание, что по мере ускорения протона его скорость остается перпендикулярной магнитному полю, поэтому величина силы не меняется.Кроме того, из-за правила правой руки направление силы остается перпендикулярным скорости. Эта сила – не что иное, как центростремительная сила: она имеет постоянную величину и всегда перпендикулярна скорости. Таким образом, величина скорости не меняется, и протон совершает круговое движение. Радиус этого круга может быть найден с помощью кинематического соотношения.

F = ma = mv2ra = v2rr = v2a = (3,0 × 106 м / с) 25,7 × 1014 м / с2 = 1,6 см F = ma = mv2ra = v2rr = v2a = (3,0 × 106 м / с) 25.7 × 1014 м / с2 = 1,6 см

20,8

Путь протона в магнитном поле показан на рисунке 20.22.

Рис. 20.22 При перемещении перпендикулярно постоянному магнитному полю заряженная частица будет совершать круговое движение, как показано здесь для протона.

Рабочий пример

Проволока с током в магнитном поле

Теперь предположим, что мы пропустили провод через однородное магнитное поле из предыдущего примера, как показано. Если по проводу проходит ток 1.-направлении, а длина области с магнитным полем 4,0 см, какова сила на проводе?

Стратегия

Используйте уравнение F = IℓBsinθF = IℓBsinθ, чтобы найти величину силы, действующей на провод. Длина провода внутри магнитного поля составляет 4,0 см, а угол между направлением тока и направлением магнитного поля составляет 90 °. Чтобы найти направление силы, используйте правило правой руки, как описано сразу после уравнения F = IℓBsinθ.F = IℓBsinθ.

Решение

Вставьте указанные значения в уравнение F = IℓBsinθF = IℓBsinθ, чтобы найти величину силы

F = IℓBsinθ = (1.5A) (0,040 м) (2,0T) = 0,12N. F = IℓBsinθ = (1,5A) (0,040 м) (2,0T) = 0,12N.

20,9

Чтобы найти направление силы, начните с размещения вектора тока вплотную к вектору магнитного поля. Результат показан на рисунке в предыдущем рабочем примере с заменой v → v → на I → I →. Согните пальцы правой руки от I → I → к B → B →, а большой палец правой руки указывает вниз по странице, как показано на рисунке в предыдущем рабочем примере..-направление. Сила, действующая на провод с током в магнитном поле, является основой всех электродвигателей, как мы увидим в следующих разделах.

Практические задачи

1.

Какова величина силы, действующей на электрон, движущийся со скоростью 1,0 × 106 м / с перпендикулярно магнитному полю 1,0 Тл?

  1. 0,8 × 10 –13 N
  2. 1,6 × 10 –14 N
  3. 0,8 × 10 –14 N
  4. 1,6 × 10 –13 N
2.

Прямой 10-сантиметровый провод имеет ток 0,40 А и ориентирован перпендикулярно магнитному полю. Если сила на проводе 0,022 Н, какова величина магнитного поля?

  1. 1,10 × 10 –2 T
  2. 0,55 × 10 –2 T
  3. 1,10 т
  4. 0,55 т

Проверьте свое понимание

3.

Если два магнита отталкиваются друг от друга, какой можно сделать вывод об их взаимной ориентации?

  1. Либо южный полюс магнита 1 ближе к северному полюсу магнита 2, либо северный полюс магнита 1 ближе к южному полюсу магнита 2.
  2. Либо южные полюса магнита 1 и магнита 2 ближе друг к другу, либо северные полюса магнита 1 и магнита 2 расположены ближе друг к другу.
4.

Опишите методы размагничивания ферромагнетика.

  1. путем охлаждения, нагрева или погружения в воду
  2. путем нагревания, удара и вращения во внешнем магнитном поле
  3. молотком, нагреванием и протиранием тканью
  4. путем охлаждения, погружения в воду или протирания тканью
5.

Что такое магнитное поле?

  1. Направляющие линии внутри и снаружи магнитного материала, указывающие величину и направление магнитной силы.
  2. Направляющие линии внутри и снаружи магнитного материала, указывающие величину магнитной силы.
  3. Направляющие линии внутри магнитного материала, указывающие величину и направление магнитной силы.
  4. Направляющие линии вне магнитного материала указывают величину и направление магнитной силы.
6.

Какой из следующих рисунков правильный?

Магнитное поле Земли

Магнитосфера защищает поверхность Земли от заряженных частиц солнечного ветра и генерируется электрическими токами, расположенными во многих различных частях Земли.Он сжимается на дневной (солнечной) стороне за счет силы приходящих частиц и расширяется на ночной стороне. (Изображение не в масштабе.) Разница между магнитным севером и «истинным» севером.

Магнитное поле Земли (и поверхностное магнитное поле ) приблизительно представляет собой магнитный диполь с S-полюсом магнитного поля вблизи географического северного полюса Земли (см. Магнитный северный полюс) и другим северным полюсом магнитного поля вблизи географического географического полюса Земли. южный полюс (см. Южный магнитный полюс).Благодаря этому компас можно использовать для навигации. Причину возникновения поля можно объяснить теорией динамо. Магнитное поле распространяется бесконечно, но ослабевает по мере удаления от источника. Магнитное поле Земли, также называемое геомагнитным полем , которое эффективно распространяется на несколько десятков тысяч километров в космос, формирует магнитосферу Земли. Палеомагнитное исследование австралийского красного дацита и подушечного базальта оценило возраст магнитного поля как минимум 3,5 миллиарда лет. [1] [2]

Важность

Моделирование взаимодействия между магнитным полем Земли и межпланетным магнитным полем.

Земля в значительной степени защищена от солнечного ветра, потока энергичных заряженных частиц, исходящих от Солнца, своим магнитным полем, которое отклоняет большинство заряженных частиц. Некоторые из заряженных частиц солнечного ветра захвачены в радиационном поясе Ван Аллена.Меньшему количеству частиц солнечного ветра удается перемещаться, как по линии передачи электромагнитной энергии, в верхние слои атмосферы и ионосферу Земли в зонах полярных сияний. Единственный раз, когда солнечный ветер наблюдается на Земле, – это когда он достаточно силен, чтобы вызывать такие явления, как полярное сияние и геомагнитные бури. Яркие полярные сияния сильно нагревают ионосферу, заставляя ее плазму расширяться в магнитосферу, увеличивая размер плазменной геосферы и вызывая утечку атмосферного вещества в солнечный ветер.Геомагнитные бури возникают, когда давление плазмы, содержащейся внутри магнитосферы, достаточно велико, чтобы раздуваться и тем самым искажать геомагнитное поле.

Солнечный ветер отвечает за общую форму магнитосферы Земли, и колебания ее скорости, плотности, направления и увлекаемого магнитного поля сильно влияют на локальную космическую среду Земли. Например, уровни ионизирующего излучения и радиопомех могут варьироваться от сотен до тысяч раз; а форма и расположение магнитопаузы и головной ударной волны перед ней могут изменяться на несколько радиусов Земли, подвергая геосинхронные спутники воздействию прямого солнечного ветра.Эти явления собирательно называются космической погодой. Механизм атмосферного разрыва вызван захватом газа пузырьками магнитного поля, которые срываются солнечными ветрами. [3] Изменения напряженности магнитного поля коррелировали с изменением количества осадков в тропиках. [4]

Магнитные полюса и магнитный диполь

Основные статьи: Северный магнитный полюс и Южный магнитный полюс Магнитное склонение от истинного севера в 1700

Положения магнитных полюсов можно определить как минимум двумя способами [5] .

Часто магнитный (наклонный) полюс рассматривается как точка на поверхности Земли, где магнитное поле полностью вертикально. Другими словами, угол наклона поля Земли составляет 90 ° на северном магнитном полюсе и -90 ° на южном магнитном полюсе. На магнитном полюсе компас, удерживаемый в горизонтальной плоскости, указывает случайным образом, в то время как в противном случае он указывает почти на северный магнитный полюс или от Южного магнитного полюса, хотя существуют местные отклонения. Два полюса перемещаются независимо друг от друга и не находятся в прямо противоположных положениях на земном шаре.Магнитный полюс падения может быстро перемещаться, наблюдения до 40 км в год были сделаны для Северного магнитного полюса [6] .

Магнитное поле Земли можно точно описать полем магнитного диполя, расположенного рядом с центром Земли. Ориентация диполя определяется осью. Два положения, в которых ось диполя, которая лучше всего соответствует геомагнитному полю, пересекает поверхность Земли, называются Северным и Южным геомагнитными полюсами. Для наилучшего соответствия диполь, представляющий геомагнитное поле, должен быть размещен примерно в 500 км от центра Земли.Это заставляет внутренний радиационный пояс опускаться ниже в южной части Атлантического океана, где поверхностное поле является самым слабым, создавая то, что называется южноатлантической аномалией.

Если бы магнитное поле Земли было идеально дипольным, геомагнитный и магнитный полюса падения совпадали. Однако важные недиполярные члены в точном описании геомагнитного поля приводят к тому, что положения двух типов полюсов находятся в разных местах.

Характеристики поля

Напряженность поля на поверхности Земли составляет менее 30 микротесла (0.3 гаусса) на территории, включающей большую часть Южной Америки и Южной Африки, до более чем 60 микротеслов (0,6 гаусса) вокруг магнитных полюсов в северной Канаде и на юге Австралии, а также в части Сибири. Средняя напряженность магнитного поля во внешнем ядре Земли составила 25 Гаусс, что в 50 раз сильнее, чем магнитное поле на поверхности. [9] [10]

Поле аналогично полю стержневого магнита. Магнитное поле Земли в основном вызвано электрическими токами в жидком внешнем ядре.Ядро Земли горячее, чем 1043 К, температура точки Кюри, выше которой ориентация спинов в железе становится случайной. Такая рандомизация приводит к потере намагниченности вещества.

Конвекция расплавленного железа во внешнем жидком ядре, наряду с эффектом Кориолиса, вызванным общим вращением планеты, имеет тенденцию организовывать эти «электрические токи» в валки, выровненные вдоль полярной оси север-юг. Когда проводящая жидкость течет через существующее магнитное поле, индуцируются электрические токи, которые, в свою очередь, создают другое магнитное поле.Когда это магнитное поле усиливает исходное магнитное поле, создается динамо-машина, которая поддерживает себя. Это называется теорией динамо, и она объясняет, как поддерживается магнитное поле Земли.

Еще одна особенность, которая магнитно отличает Землю от стержневого магнита, – это ее магнитосфера. На больших расстояниях от планеты преобладает поверхностное магнитное поле. Электрические токи, индуцированные в ионосфере, также создают магнитные поля. Такое поле всегда создается вблизи того места, где атмосфера находится ближе всего к Солнцу, вызывая ежедневные изменения, которые могут отклонять поверхностные магнитные поля на величину до одного градуса.Типичные ежедневные изменения напряженности поля составляют около 25 нанотесла (нТл) (т.е. ~ 1: 2 000), с вариациями в течение нескольких секунд, как правило, около 1 нТл (т.е. ~ 1: 50 000). [11]

Вариации магнитного поля

Геомагнитные вариации с момента последнего обращения.

Токи в ядре Земли, создающие ее магнитное поле, возникли по крайней мере 3 450 миллионов лет назад. [12] [13]

Магнитометры обнаруживают мельчайшие отклонения в магнитном поле Земли, вызванные железными артефактами, печами, некоторыми типами каменных построек и даже канавами и мусором в археологической геофизике.С помощью магнитных инструментов, адаптированных на основе бортовых детекторов магнитных аномалий, разработанных во время Второй мировой войны для обнаружения подводных лодок, были нанесены на карту магнитные вариации на дне океана. Базальт – богатая железом вулканическая порода, составляющая дно океана – содержит сильно магнитный минерал (магнетит) и может локально искажать показания компаса. Искажение было признано исландскими мореплавателями еще в конце 18 века. Что еще более важно, поскольку присутствие магнетита придает базальту измеримые магнитные свойства, эти магнитные вариации предоставили еще один способ изучения глубоководного дна океана.Когда вновь образованная порода охлаждается, такие магнитные материалы регистрируют магнитное поле Земли.

Часто магнитосфера Земли поражается солнечными вспышками, вызывающими геомагнитные бури, вызывающие проявления полярных сияний. Кратковременная нестабильность магнитного поля измеряется с помощью K-индекса.

Недавно в магнитном поле были обнаружены утечки, которые взаимодействуют с солнечным ветром Солнца способом, противоположным первоначальной гипотезе. Во время солнечных бурь это может привести к крупномасштабным отключениям электроэнергии и сбоям в работе искусственных спутников. [14]

См. Также Магнитная аномалия

Инверсия магнитного поля

Основная статья: Геомагнитная инверсия

Основываясь на изучении лавовых потоков базальта во всем мире, было высказано предположение, что магнитное поле Земли меняет направление на противоположное. с интервалами от десятков тысяч до многих миллионов лет, со средним интервалом примерно 300 000 лет. [15] Однако последнее такое событие, называемое инверсией Брюнес – Матуяма, произошло примерно 780 000 лет назад.

Нет четкой теории относительно того, как могли произойти геомагнитные инверсии. Некоторые ученые создали модели ядра Земли, в которых магнитное поле лишь квазистабильно, а полюса могут самопроизвольно перемещаться из одной ориентации в другую в течение от нескольких сотен до нескольких тысяч лет. Другие ученые предполагают, что геодинамо сначала отключается самопроизвольно или из-за какого-то внешнего воздействия, такого как удар кометы, а затем перезапускается с магнитным «северным» полюсом, указывающим либо на север, либо на юг.Внешние события вряд ли будут обычными причинами инверсий магнитного поля из-за отсутствия корреляции между возрастом ударных кратеров и временем инверсий. Независимо от причины, когда магнитный полюс переключается из одного полушария в другое, это называется инверсией, тогда как временные изменения наклона диполя, которые перемещают ось диполя через экватор, а затем возвращаются к исходной полярности, известны как отклонения.

Исследования потоков лавы на горе Стинс, штат Орегон, показывают, что магнитное поле могло смещаться со скоростью до 6 градусов в день в какой-то момент истории Земли, что значительно бросает вызов популярному пониманию того, как работает магнитное поле Земли. . [16]

Палеомагнитные исследования, подобные этим, обычно состоят из измерений остаточной намагниченности вулканических пород. Осадки, отложенные на дне океана, ориентируются в соответствии с местным магнитным полем, сигнал, который может быть записан по мере их затвердевания. Хотя залежи магматических пород в основном парамагнитны, они действительно содержат следы ферри- и антиферромагнитных материалов в виде оксидов железа, что дает им способность обладать остаточной намагниченностью.Фактически, эта характеристика довольно часто встречается во многих других типах горных пород и отложений, обнаруженных по всему миру. Магнетит – один из наиболее распространенных оксидов, обнаруживаемых в природных отложениях.

В качестве примера того, как это свойство магматических пород позволяет нам определить, что поле Земли в прошлом менялось, рассмотрим измерения магнетизма на океанских хребтах. Прежде чем магма выйдет из мантии через трещину, она имеет чрезвычайно высокую температуру, превышающую температуру Кюри любого закиси железа, который она может содержать.Лава начинает остывать и затвердевать, когда попадает в океан, позволяя этим оксидам железа в конечном итоге восстановить свои магнитные свойства, в частности, способность удерживать остаточную намагниченность. Если предположить, что единственное магнитное поле, присутствующее в этих местах, связано с самой Землей, эта затвердевшая порода становится намагниченной в направлении геомагнитного поля. Несмотря на то, что напряженность поля довольно мала, а содержание железа в типичных образцах горных пород невелико, относительно небольшая остаточная намагниченность образцов находится в пределах разрешающей способности современных магнитометров.Затем можно измерить возраст и намагниченность застывших образцов лавы, чтобы определить ориентацию геомагнитного поля в древние эпохи.

Обнаружение магнитного поля

Отклонения модели магнитного поля от данных измерений, данных, созданных спутниками с чувствительными магнитометрами

Напряженность магнитного поля Земли была измерена Карлом Фридрихом Гауссом в 1835 году и с тех пор неоднократно измерялась, показывая относительное затухание около 10% за последние 150 лет. [17] Спутник Magsat и более поздние спутники использовали 3-осевые векторные магнитометры для исследования трехмерной структуры магнитного поля Земли. Более поздний спутник Эрстеда позволил провести сравнение, показывающее динамическое геодинамо в действии, которое, по-видимому, порождает альтернативный полюс под Атлантическим океаном к западу от Южной Африки. [18]

Правительства иногда используют подразделения, специализирующиеся на измерении магнитного поля Земли. Это геомагнитные обсерватории, обычно входящие в состав национальной геологической службы, например, обсерватория Эскдалемуир Британской геологической службы.Такие обсерватории могут измерять и прогнозировать магнитные условия, которые иногда влияют на связь, электроэнергию и другую деятельность человека. (См. Магнитную бурю.)

Международная сеть магнитных обсерваторий в реальном времени с более чем 100 взаимосвязанными геомагнитными обсерваториями по всему миру с 1991 года регистрирует магнитное поле Земли.

Военные определяют местные характеристики геомагнитного поля по порядку. для обнаружения аномалий на естественном фоне, которые могут быть вызваны значительным металлическим объектом, например, затопленной подводной лодкой.Как правило, эти детекторы магнитных аномалий используются в самолетах, таких как британский Nimrod, или буксируются в качестве инструмента или набора инструментов с надводных кораблей.

В коммерческих целях геофизические разведочные компании также используют магнитные детекторы для выявления естественных аномалий рудных тел, таких как Курская магнитная аномалия.

Животные, включая птиц и черепах, могут обнаруживать магнитное поле Земли и использовать это поле для навигации во время миграции. [19] Коровы и дикие олени склонны выстраивать свои тела с севера на юг во время отдыха, но не тогда, когда животные находятся под высоковольтными линиями электропередач, что заставляет исследователей полагать, что причиной этого является магнетизм. Дайсон, П.Дж. (2009). «Биология: электрические коровы». Природа 458 (7237): 389. DOI: 10.1038 / 458389a. PMID 19325587.

Внешние ссылки


  • Уильям Дж. Брод, Будет ли компас указывать на юг? . New York Times, 13 июля 2004 г.
  • John Roach, Почему магнитное поле Земли меняется? . National Geographic, 27 сентября 2004 г.
  • Когда север идет на юг . Проекты в области научных вычислений, 1996.
  • Трехмерный имитатор заряженных частиц в магнитном поле Земли . Инструмент, предназначенный для трехмерного моделирования заряженных частиц в магнитосфере. [Требуется подключаемый модуль VRML]
  • Великий Магнит, Земля , История открытия магнитного поля Земли Дэвидом П. Стерном.
  • Исследование магнитосферы Земли , Образовательный веб-сайт Дэвида П. Стерна и Маурисио Передо

магнетизм | Национальное географическое общество

Магнетизм – это сила, проявляемая магнитами, когда они притягиваются или отталкиваются друг от друга.Магнетизм вызывается движением электрических зарядов.

Каждое вещество состоит из крошечных единиц, называемых атомами. В каждом атоме есть электроны, частицы, несущие электрические заряды. Вращаясь, как волчки, электроны вращаются вокруг ядра или остова атома. Их движение генерирует электрический ток и заставляет каждый электрон действовать как микроскопический магнит.

В большинстве веществ равное количество электронов вращается в противоположных направлениях, что нейтрализует их магнетизм. Вот почему такие материалы, как ткань или бумага, считаются слабомагнитными.В таких веществах, как железо, кобальт и никель, большинство электронов вращаются в одном направлении. Это делает атомы в этих веществах сильно магнитными, но они еще не магниты.

Чтобы стать намагниченным, другое сильномагнитное вещество должно войти в магнитное поле существующего магнита. Магнитное поле – это область вокруг магнита, обладающая магнитной силой.

Все магниты имеют северный и южный полюса. Противоположные полюса притягиваются друг к другу, а одни и те же полюса отталкиваются.Когда вы протираете кусок железа по магниту, северные полюса атомов в железе выстраиваются в одном направлении. Сила, создаваемая выровненными атомами, создает магнитное поле. Железка стала магнитом.

Некоторые вещества могут намагничиваться электрическим током. Когда электричество проходит через катушку с проволокой, создается магнитное поле. Однако поле вокруг катушки исчезнет, ​​как только отключится электрический ток.

Геомагнитные полюса

Земля – ​​это магнит.Ученые не до конца понимают, почему, но они думают, что движение расплавленного металла во внешнем ядре Земли порождает электрические токи. Токи создают магнитное поле с невидимыми силовыми линиями, протекающими между магнитными полюсами Земли.

Геомагнитные полюса не совпадают с Северным и Южным полюсами. Магнитные полюса Земли часто перемещаются из-за активности далеко под поверхностью Земли. Смещение геомагнитных полюсов регистрируется в породах, которые образуются, когда расплавленный материал, называемый магмой, проникает сквозь земную кору и изливается в виде лавы.Когда лава остывает и превращается в твердую породу, сильно магнитные частицы внутри породы намагничиваются магнитным полем Земли. Частицы выстраиваются вдоль силовых линий в поле Земли. Таким образом, камни фиксируют положение геомагнитных полюсов Земли в то время.

Как ни странно, магнитные записи горных пород, образовавшихся в одно и то же время, похоже, указывают на разные местоположения полюсов. Согласно теории тектоники плит, скальные плиты, составляющие твердую оболочку Земли, постоянно перемещаются.Таким образом, плиты, на которых застывала порода, переместились с тех пор, как породы зафиксировали положение геомагнитных полюсов. Эти магнитные записи также показывают, что геомагнитные полюса менялись местами – превращались в полюсы противоположного типа – сотни раз с момента образования Земли.

Магнитное поле Земли не движется быстро и часто не меняется. Следовательно, это может быть полезным инструментом, помогающим людям сориентироваться. Сотни лет люди использовали магнитные компасы для навигации по магнитному полю Земли.Магнитная стрелка компаса совпадает с магнитными полюсами Земли. Северный конец магнита указывает на северный магнитный полюс.

Магнитное поле Земли доминирует в области, называемой магнитосферой, которая охватывает планету и ее атмосферу. Солнечный ветер, заряженные частицы от Солнца, прижимает магнитосферу к Земле со стороны, обращенной к Солнцу, и растягивает ее в форме капли на теневой стороне.

Магнитосфера защищает Землю от большинства частиц, но некоторые из них просачиваются сквозь нее и попадают в ловушку.Когда частицы солнечного ветра сталкиваются с атомами газа в верхних слоях атмосферы вокруг геомагнитных полюсов, они создают световые эффекты, называемые полярными сияниями. Эти полярные сияния появляются над такими местами, как Аляска, Канада и Скандинавия, где их иногда называют «Северным сиянием». «Южное сияние» можно увидеть в Антарктиде и Новой Зеландии.

электромагнетизм – Как движущиеся заряды создают магнитные поля?

Если вы плохо знакомы со специальной теорией относительности, невозможно по-настоящему объяснить это явление.Лучшее, что можно сделать, – это дать вам правил , пропитанных эзотерическими идеями, такими как «электромагнитное поле» и «лоренц-инвариантность». Конечно, это не то, что вам нужно, и это правильно, поскольку физика никогда не должна сводиться к безосновательному принятию правил, установленных свыше.

Дело в том, что магнетизм – это не что иное, как электростатика в сочетании со специальной теорией относительности . К сожалению, вы не найдете много книг, объясняющих это – либо авторы ошибочно полагают, что уравнения Максвелла не имеют оправдания и должны приниматься на веру, либо они слишком погрязли в своих собственных эзотерических обозначениях, чтобы остановиться, чтобы задуматься о том, что они говорят.Единственная известная мне книга, которая правильно трактует эту тему, – это книга Перселла «Электричество и магнетизм », которая недавно была переиздана в третьем издании. (Второе издание отлично подойдет, если вы сможете найти копию.)

Краткий эвристический набросок идеи заключается в следующем. Предположим, есть линия положительных зарядов, движущаяся по оси $ z $ в положительном направлении – ток. Рассмотрим положительный заряд $ q $, расположенный в $ (x, y, z) = (1,0,0) $, движущийся в отрицательном направлении $ z $.Мы видим, что из-за всех этих зарядов на $ q $ будет некоторая электростатическая сила.

Но давайте попробуем что-нибудь безумное – давайте перейдем к системе отсчета $ q $. В конце концов, законы физики должны соблюдаться со всех точек зрения. Ясно, что заряды, составляющие ток, в этой системе координат будут двигаться быстрее. Но это мало что дает, поскольку в конце концов кулоновская сила явно не заботится о скорости зарядов, а только об их разделении. Но специальная теория относительности говорит нам о другом.2} $, где $ v $ – скорость $ q $ в исходном кадре. Это знаменитое сокращение длины, предсказанное специальной теорией относительности.

Если текущие заряды оказываются ближе друг к другу, тогда очевидно, что $ q $ будет ощущать большую электростатическую силу от оси $ z $ в целом. Он будет испытывать дополнительную силу в положительном направлении $ x $, от оси, сверх того, что мы ожидали, просто сидя в лабораторной раме. По сути, закон Кулона – это закон силы только , действующий на заряд, но только система покоя заряда действительна для использования этого закона для определения силы, которую испытывает заряд.

Вместо того, чтобы постоянно переключаться между кадрами, мы изобретаем магнитное поле как математическое устройство, которое выполняет то же самое. При правильном определении он будет полностью объяснять эту аномальную силу, по-видимому, испытываемую зарядом, когда мы наблюдаем его не в его собственной системе покоя. В примере, который я только что рассмотрел, правило правой руки говорит вам, что мы должны приписать магнитное поле току, вращающемуся вокруг оси $ z $, так, чтобы он указывал в положительном направлении $ y $ в месте расположения $ q $.Скорость заряда находится в отрицательном направлении $ z $, и поэтому $ q \ vec {v} \ times \ vec {B} $ указывает в положительном направлении $ x $, как мы узнали из изменения системы отсчета. .

9.3 Магнитное поле Земли – Физическая геология

Глава 9 Внутренние части Земли

Тепло также передается от твердого внутреннего ядра к жидкому внешнему ядру, что приводит к конвекции жидкого железа внешнего ядра. Поскольку железо является металлом и проводит электричество (даже в расплавленном состоянии), его движение создает магнитное поле.

Магнитное поле Земли определяется Северным и Южным полюсами, которые обычно совпадают с осью вращения (рис. 9.13). Магнитные силовые линии текут на Землю в северном полушарии и выходят из Земли в южном полушарии. Из-за формы силовых линий магнитная сила движется под разными углами к поверхности в разных местах (красные стрелки на рисунке 9.13). На Северном и Южном полюсах сила вертикальная. В любом месте на экваторе сила горизонтальна, а повсюду между ними магнитная сила находится под некоторым промежуточным углом к ​​поверхности.Как мы увидим в главе 10, различия в этих ориентациях являются важным свидетельством понимания дрейфа континентов как аспекта тектоники плит.

Магнитное поле Земли создается во внешнем ядре за счет конвективного движения жидкого железа, но, как мы обнаружили в главе 8, магнитное поле нестабильно в течение геологического времени. По причинам, которые до конца не изучены, магнитное поле периодически спадает, а затем восстанавливается.Когда он действительно восстанавливается, он может быть ориентирован так, как был до распада, или он может быть ориентирован с обратной полярностью. За последние 250 млн лет произошло несколько сотен инверсий магнитного поля, и их время было совсем не регулярным. Самые короткие из них, которые геологи смогли определить, длились всего несколько тысяч лет, а самые длинные – более 30 миллионов лет в меловом периоде (рис. 9.14).

Рис. 9.13. Изображение магнитного поля Земли в виде стержневого магнита, совпадающего с ядром.Южный полюс такого магнита указывает на Северный полюс Земли. Красные стрелки показывают ориентацию магнитного поля в различных местах на поверхности Земли. [SE после: http://upload.wikimedia.org/wikipedia/commons/ 1/17 / Earths_Mintage_Field_ Confusion.svg]

Упражнение 9.3 Что говорит вам ваш магнитный измеритель угла падения?

Обычные компасы указывают только на северный магнитный полюс, но если у вас есть измеритель магнитного угла наклона (или iPhone с соответствующим приложением *), вы также можете измерить угол магнитного поля в вашем местоположении в вертикальном направлении. смысл.Для выполнения этого упражнения не нужно покупать приложение (или iPhone)!

Используя рисунок 9.13 в качестве руководства, опишите, где бы вы были на Земле, если вертикальные углы будут следующими:

Вверх под небольшим углом Параллельно земле

Вертикальная ориентация Общее местонахождение Вертикальная ориентация Общее местонахождение
Прямо вниз
Вниз под крутым углом

* См. Приложение для определения магнитного наклона по адресу: http: // www.hotto.de/mobileapps/iphonemintageinclinationmeter.html

Рис. 9.14. Хронология инверсии магнитного поля за последние 170 млн лет. Первые 5 млн лет магнитной хронологии более подробно показаны на рис. 9.15. [SE после: http://upload.wikimedia.org/wikipedia/en/c/c0/Geomintage_polarity_0-169_Ma.svg]

Изменения в магнитном поле Земли были изучены с использованием математической модели, и было показано, что инверсии имеют место, когда модель использовалась для моделирования периода в несколько сотен тысяч лет.Тот факт, что произошли инверсии поля, показывает, что модель является достаточно точным представлением Земли. По словам ведущего автора исследования Гэри Глатцмайера из Калифорнийского университета в Санта-Круз: «Наше решение показывает, как конвекция во внешнем жидком ядре постоянно пытается изменить направление поля, но что твердое внутреннее ядро ​​препятствует инверсии магнитного поля, потому что поле в внутреннее ядро ​​может измениться только в гораздо более длительном временном масштабе диффузии. Только один раз из многих попыток инверсия оказывается успешной, что, вероятно, является причиной того, что времена между инверсиями поля Земли длинные и распределены случайным образом.Изображение силовых линий магнитного поля Земли в стабильный период и во время переворота показано на рисунке 9.15. Чтобы узнать больше об этих явлениях, посетите веб-сайт Глатцмайера «Геодинамо» по адресу: http://es.ucsc.edu/~glatz/geodynamo.html.

Рис. 9.15. Изображение магнитного поля Земли между инверсиями (слева) и во время инверсии (справа).

Оставить комментарий